Docsity
Docsity

Prepare for your exams
Prepare for your exams

Study with the several resources on Docsity


Earn points to download
Earn points to download

Earn points by helping other students or get them with a premium plan


Guidelines and tips
Guidelines and tips

Medical Diagnosis and Treatment Information, Exams of Nursing

Various medical diagnosis and treatment information, including types of hemophilia, canine oral fluids organisms, complications of certain diseases, lab report review preparation, spriometry readings, symptoms of rheumatic fever and measles, sequelae of gabhs infection, diagnostic criteria for acute rheumatic fever, treatment options for utis, olecranon bursitis causes, oral contraceptive regimens, febrile seizures prevention, neuroblastoma diagnosis, antibiotics of choice for different infections, and knee locking causes. It also includes some client teaching techniques and observations for potential complications.

Typology: Exams

2023/2024

Available from 02/21/2024

experttutor001
experttutor001 🇺🇸

3

(4)

427 documents

1 / 101

Toggle sidebar

Related documents


Partial preview of the text

Download Medical Diagnosis and Treatment Information and more Exams Nursing in PDF only on Docsity! APEA Predictor with complete answers latest updated 2024 Cranial Nerve VIII - correct answerAcoustic (Hearing and balance) Cranial nerve IX - correct answerGlossopharyngeal- swallowing and rise of the palate, gag reflex Cranial Nerve XI - correct answerSpinal Accessory (upper trapezius) Deep tendon reflex - correct answerAnkle- S1 primarily Knee- Lumbar 2,3,4 Supinator(brachioradialis)-C5,6 BicepTriceps-C6,7 Stroke risk scale - correct answerCHADS2 C-Congestive heart failure H-Hypertension A-Age >75 years D- Diabetes S-prior Stroke/TIA In the majority of children, the first permanent teeth start to erupt at the age of 6 years. Which of the following are the first permanent teeth to erupt in this time period? A. First Molars B. Second Molars C. Lower or upper incisors D. Canines - correct answerA. First Molars All of the following are classified as activities of daily living (ADLs) except: A. ability to feed self B. Ability to manage bladder and bowel elimination C. Personal hygiene and grooming D. Grocery Shopping - correct answerD. Grocery shopping You note the following result on a routine urinalysis of a 37-year old primigravida who is at 30 weeks gestation. Leukocyte=trace, nitrite=negative, protein=2+, blood=negative. Her weight has increased by 5 lbs during the past week. Which of the following is most likely? A.HELLP syndrome B. Pregnancy-induced hypertension (pre-eclampsia) C. Eclampsia of pregnancy D. Primary hypertension - correct answerB. Pregnancy-induced hypertension (pre- eclampsia). Classic triad of symptoms of preeclampsia include hypertension, edema (weight gain), and proteinuria. The mother of a 12 month old infant reports to the nurse practitioner that her child had a high fever for several days, which spontaneously resolved. After the fever resolved, the APEA Predictor with complete answers latest updated 2024 child developed a maculopapular rash. Which of the following is the most likely diagnosis? A. Fifth Disease (erythema infectiosum) B. Roseola infantum (exanthema subitum) C. Varicella D. Infantile maculopapular rashes - correct answerB. Roseola Infantum (Exanthema subitum) A 70-year old male patient complains of a bright red-colored spot in his left eye for 2 days. He denies eye pain, visual changes, or headaches. He has a new onset of cough from a recent viral upper respiratory infection. The only medicine he is on Bayer aspirin, 1 tablet a day. Which of the following is most likely? A. Corneal abrasion B. Acute bacterial conjunctivitis C. Acute uveitis D. Subconjunctival hemorrhage - correct answerD. Subconjunctival hemorrhage. Bright red blood in a sharply defined area surrounded by normal-appearing conjunctiva indicates subconjunctival hemorrhage. A woman is being evaluated by the nurse practitioner for complaints of dyspareunia. A microscopy slide reveals a large number of atrophic squamous epithelial cells. The vaginal pH is 4.0. There are very few leukocytes and no RBCs are seen on the wet smear. Which of the following is most likely? A. Atrophic vaginitis B. Bacterial Vaginosis C. Trichomoniasis D. This is a normal finding. - correct answerA. Atrophic vaginitis. Symptoms of atrophic vaginitis include painful intercourse, atrophic squamous epithelial cells, and a decrease in pH. Vaginal atrophy is caused by lack of or imbalance of estrogen. Normal pH of the vagina is 4.0-5.0 (acidic) The most common type of skin malignancy is: A. Squamous cell cancer B. Basal skin cancer C. Melanoma D. Dysplastic nevi - correct answerB. Basal skin cancer. It is several times more common that squamous cell skin cancer according to the NIH. Koplik's spots are associated with: A. Poxvirius infections B. Measles C. Kawasaki's disease D. Reye's syndrome - correct answerB. Measles. S/S include fever over 101, coryza, cough, conjunctivitis, rash, and koplik's spots on buccal mucosa APEA Predictor with complete answers latest updated 2024 positive Murphy's inspriatory arrest. Ilcerative colitis presents with LLQ pain. Pain associated with pancreatitis is typically in the epigastric region. Atenolol (Tenormin) should be avoided in: - correct answera 43-yo female with asthma. Atenolol is a beta blocker. It should be avoided in pateints with asthma or other bronchospastic conditions. These patients often require beta stimulation, not beta blockade. Although atenolol is a beta 1-selective agent, at higher doses beta 2 receptors (bronchial and vascular) are blocked. Beta blockers, specifically atenolol, are beneficial post-myocardial infarction because they have demonstrated a reduction in morbidity and mortality. All of the following interventions with pediatric patients are appropriate EXCEPT: - correct answerPre-medicate the patient prior to all painful interventions. Pre-medication is not warranted prior to all painful interventions. For example, premedication is not warranted prior to routine immunizations but it certainly is prior to suturing. Pain management should be an integral part of patient management. Infants with veliac disease (gluten eneteropathy) are at risk for multiple complications. The most urgen complication of this disease is: - correct answerIntususseption or volvulus. Intuseusseption and volvulus are surgical emergencies. Delay releasing the invaginated or "telescoped" bowel (intususseption), or releasing the twisted bowel (volvulus) may result in tissue death and gangrene, perforation, peritonitis, and/or sepsis, and fatality. There is also a high rate of intususseption and volvulus among infants with cistic fibrosis. Which activities are NOT characteristic of preschool children? - correct answerAlways follow rules during playground games. Since preschoolers are just beginning to learn moral behaviors, they often cheat to win. While most preschoolers toilet independently, accidents occasionally occur and bed- wetting is not unusual. The use of a security item such as a blanket is common. A 38-yo pregnant patient at 18 weeks gestation, complains of feeling light-headed when standing. Which of the following is an appropriate response by the NP? - correct answerBlood pressure normally decreases during pregnancy and can cause this symptom. Blood pressure normally decreases during pregnancy, reaching the lowest point during the second or third trimesters and rising there after. Patient education to rise slowly from sitting or lying is important. Low blood glucose may be the etiology, but an oral glucose tolerance test at this point is not indicated. A fasting blood glucose could be ordered, however, an electrocardiogram is not indicated. An 80 year old Caucasian female has heart failure. What symptom is an early indicator of failure? - correct answerWeight gain. APEA Predictor with complete answers latest updated 2024 Early signs of heart failure include weight gain (the most sensitive indicator), S3 gallop, dyspnea on exertion, peripheral edema. Moderate signs include nocturnal cough, tachycardia. Late signs include ascities, frothy sputum, and hypotension. A 6 yo had an acute onset of fever, pharyngitis, and headache 2 days ago. Today, he presents with cervical lymphadenopathy and sandpaper textured rash everywhere except on his face. A rapid streptococcal antigen test is positive. The remainder of the assessment in unremarkable. What is the most likely diagnosis and the most appropriate action? - correct answerScarlet fever; treat with antibiotics. This disease is due to infection with Group A Beta-hemolytic streptococcus. The rash is thought to be due to a systemic reaction to the toxin produced by the microorganism. The rash fades with pressure and ultimately desquamates. A deep, nonblanching rash on the flexor surfaces of the skin is referred to as pastia lines. A patient has been diagnosed with hypothyroidism and thyroid hormone replacement therapy is prescribed. When should the nurse practitioner check the patient's TSH? - correct answer6 weeks. The half-life of levothyroxine, the treatment of choice for thyroid replacement, is 7 days. The earliest that meaningful changes will be observed is at 4-6 weeks. Therefore, the NP should wait a minimum of 4-6 weeks before checking the patient's TSH. A 15 yo malue has a history of cryptorchidism which was surgically repaired. Because of this information, it is essential for the nurse practitioner to teach him about: - correct answertesticular self-examination. Cryptorchidism, even with surgical repair, is associated with increased risk for testicular cancer. The treatment of choice for chronic bacterial prostatitis (CBP) is: - correct answera flouroquinolone twice daily for 3 weeks to 4 months. The treatment of chice is a flouroquinolone twice daily for 3 weeks to 4 months. The cure rate with Bactrim-DS is only about 30-40%. A 25 yo female has a history of frequent candidal vaginal infections in the past year. She is in a monogamous sexual relationship and uses and IUD for contraception. Of the following, which is the most likely underlying conidition predisposing her to recurring candidal vaginitis? - correct answerDiabetes. A common underlying cause of frequent infections is diabetes mellitus. Pregnancy increases the incidence of candidiasis, but is unlikely a factor with this patient. Which of the following is NOT a characteristic of the S3 heart sound? - correct answerThe sound is high-pitched and occurs just prior to the S1 heart sound. The S3 heart sound is low-pitched and occurs just after the S2 heart sound. It is produced by rapid ventricular filling and is best auscultated in the mitral area. It is a APEA Predictor with complete answers latest updated 2024 common finding with right-sided heart failure, rapid growth, and the last trimester of pregnancy. Following the finding of prostate gland abnormalities on DRE, the NP orders the appropriate labs. Whem preparing to review lab reports with the patient, the nurse practitioner knows all of the following are true EXCEPT: - correct answernormal PSA is 10ng/ml or less. Normal PSA is 4ng/ml or less. PSA levels greater the 4 and less than 10 are associated with BPH. A 10 or greater PSA level suggests prostate cancer. Positive serum acid phosphatase is associated with malignancy of the prostate gland with bone metasasis. A 66 yo female presents to your clinic. She states that yesterday evening she had chest pain for 20-30 minutes. Which finding most strongly correlates with myocardial infarction? - correct answerElevated Troponin I levels An elevated creatinine kinase (CK) is not diagnostic of a myocardial infarction (MI). CK may be elevated from an IM injection, surgery, ot any type of extensive skeletal muscle trauma or prolonged, strenuous physical exertion. ST segment depression on EKG usually indicates an ischemic myocardium, but, not necessarily, one post-MI. Elevated ST seghments reflect mycardial damage. MB bands are specific for myocardial smooth muscle. If these are elevated, the patient MAY HAVE had a very recent MI. The most accurate marker of cardiac damage, because it is more specific and sensitive than CK MB, is a troponin measurement. What is a secondary cause of hyperlipidemia? - correct answerhypothyroidism Hypothyroidism is a common secondary cause of hyperlipidemia. In the evaluation of a patient with hyperlipidemia, a TSH should always be checked and corrected before attempting treatment for hyperlipidemia. Other possible causes of seconday hyperlipidemia include pregnancy, excessive weight gain, excessive alcohol intake, insulin resistance or deficiency, obstructive liver disease, and uremia. Some medications can produce secondary hypothyroidism too: thiazide diuretics, some beta- blockers, oral contraceptives, and corticosteroids. A 35-yo male presents with a complaint of low pelvic pain, dysuria, hesitancy, urgency, and reduced for of stream. The nurse practitioner suspects acute bacterial prostatitis. The NP would appropriately collect all of the following specimens EXCEPT a: sterile in- and-out catheter urine specimen. - correct answerA sterile in-and-out catheter specimen would identify only organisms in the bladder and would not differentiate between bladder, kidney, or prostate site infection. The sequence for obtaining specimens when prostat infection is suspected is: 1. voided urethral urine, 2. Voided mid-stream bladder urine, and 3. voided post-prostate massage urine. A 24 yo female taking an oral contraceptive has missed her last 2 pills. What should the nurse practitioner advise her to do to minimize her risk of pregnancy? - correct answerDouble today's dose and tomorrow's dose and use a barrier method for the rest of the month. APEA Predictor with complete answers latest updated 2024 The reason beta-adrenergic blockers should be avoided in patients with diabetes is because they may: - correct answerMask symptoms of hypoglycemia. Beta blockers may mask the peripheral signs of hypoglycemia like jitteriness and tachycardia. However, beta blockers will not mask diaphoresis. Therefore diabetics on betablockers should be taught to look for this specific symptom as a possible indication of hypoglycemia. An adult female patient is seeking information about her ideal weight. She is 5 feet 7 inches tall. Using the "height-weight formula" what is her ideal body weight? - correct answer135 lbs The height-weight formula is a quick method of determining ideal weight. Females allow 100 lbs for the first 5 feet of height plus 5 lbs for each additional inch. Males allow 106 lbs for the first 5 feet plus 6 lbs for each additional inch. This method can only be used as an estimate because it does not account for body composition or age. The nurs practitioner is caring for a 19 year old female college student with iron deficiency anemia secondary to heavy menstrual bleeding. An appropriate INITIAL treatment for this patient is: - correct answerOral ferrous sulfate. With iron deficiency anemia, iron stores of the body must be replenished as well as the underlying cause corrected. A daily iron supplement is used initially. The most common form is ferrous sulfate. Intramuscular iron dextran is usually not needed, but may be required in the presence of a malabsorption disorder, inflammatory bowel disease, intolerance to oral iron, or blood loss too great to be compensated by oral iron. A male patient with chronic atrial fibrillation takes a generic brand of Coumadin (warfarin). He should report all of these to his health care provider EXCEPT: - correct answerone missed dose of warfarin. This patient takes warfarin for prevention of emboli secondary to chronic atrial fibrillation. According to the NHLBI and ACCP, warfarin is the standard of care when anticoagulation is required for this condition. Warfarin is a drug with a narrow therapeutic index. This means that fluctuation in its level (increased or decreased) can potentiall cause big changes in its therapeutic effect. Consequently, warfarin levels are checked frequently to maintain therapeutic levels. If changes such as medication substitutions occur, warfarin levels should be checked in 3-7 days. Levels should be checked minimally every 4-6 weeks once regulation has occurred. While it is important NOT to omit doses of warfarin, this is the least important of all the choices because the effect of warfarin lasts beyond 24 hours. Which of the following medication or medication class that does NOT have dizziness or vertigo as potential adverse side effects? - correct answerMeclizine (Antivert) Dizziness is associated with a sensation of body movement when there is no body movement occurring. (The person is spinning and the room is not). Vertigo is the sensation that the person is still and the room is spinning. There is no associated muscle weakeness or visual disturbance with either of these conditions. The most APEA Predictor with complete answers latest updated 2024 common causes are related to drug ingestion, hypotension, inner or middle ear pathology, and positional vertigo. Dizziness is an adverse reaction associated with certain antibiotics (gentomycin and strptomycin) and high-dose salicylates. Vertigo is associated with certain inner ear pathology such as labyrinthitis and Meniere's disease. Meclizine is a long-acting antihistamine which is used to treat chronic vertigo. A PPD is considered positive at 5 millimeters or more for which population? - correct answerConfirmed or suspected HIV infection, injecting drug users, close contacts of a TB case, persons with a chest X-ray suggestive of TB A PPD greater than or equal to 10 mm is considered positive for injecting drug users known to be HIV negative, occupants of long-term care facilities, age less than 4 years, groups with a high prevalance for TB, the medically underserved, and healthcare workers. A PPD is considered positive at 15mm or more in those with no known risk factors. Considering mortality statistics for the adolescent age group. education targeted toward this group should first focus on: - correct answeralcohol abuse. Alcohol is the most commonly used psychoactive substance in the United States today. It has been used by about 90% of adolescents by 16 years of age. Motor vehicle accidents related to driving under the influence of alcohol are the leading cause of death in the 15-24 year old age group. A patient with moderate persistent asthma will probably be most effectively managed with daily: - correct answerinhaled steroids and long acting bronchodilators. A patient with moderate persistent asthma has symptoms daily. He is best managed with daily medications of inhaled corticosteroids and long acting bronchdilators. Oral leukotriene blockers may be added to this regimen. A child has just scalded her index finger with hot water at home. The mother calls the NP within 5 minutes of the injury. All of the following are appropiate instructions for the mother regarding the care of the patient with a 2nd degree burn EXCEPT: - correct answerApplying butter, cooking oil, or lanolin for pain relief. Flushing first or second degree burn with cool water is appropriate to prevent further thermal injury and to provide pain relief. Oil should NEVER be applied to a burn injury. Consider consultation for burns in patients who are under 10 years of age and over 50 years of age. Physician referral is recommended for all 3rd degree burns, for 2nd and 3rd degree burns involving more than 10% of the body surface area, any deep thickness burns involving more than 2% of the body surface area, and burns involving the face. What is the recommended treatment for cat scratch disease? - correct answerusually self-limiting and presents with lymphadenopathy that goes away on it's own; severe cases may benefit from azithromycin What is the most common cause of traveler's diarrhea? - correct answere coli APEA Predictor with complete answers latest updated 2024 What bacteria is responsible for causing cat scratch disease? - correct answerbartonella henselae What are the 4 different components of tetralogy of fallot? - correct answeroverriding aorta, pulmonary stenosis, right ventricular hypertrophy, and a ventricular septal defect What does the murmur auscultated in tetralogy of fallot sound like? - correct answersystolic murmur best heard at the left upper sternal border What does the murmur auscultated in a ventricular septal defect sound like? - correct answersystolic murmur best heard at the left lower sternal border Where is the aorta best auscultated? - correct answerat the second ICS to to the right of the sternum Where is the pulmonary artery best auscultated? - correct answerat the second ICS to the left of the sternum What is a scotoma? - correct answera blind spot or aura that obstructs part of your vision Is presbyopia nearsightedness or farsightedness? - correct answerfarsightedness meaning it can be harder to see things up close, such as reading a newspaper and may need to hold it farther away or use reading glasses Is myopia nearsightedness or farsightedness? - correct answernearsightedness meaning it is hard to see things far away Do cataracts cause myopia or presbyopia? - correct answermyopia; meaning nearsightedness and they make things blurry that are farther away which is why driving at night becomes difficult What is the USPSTF's recommendation on scoliosis screening for adolescents? - correct answerinsufficient When should adolescent girls be screened for scoliosis? - correct answeras early as 10 years old and then again at age 12 When should adolescent boys be screening for scoliosis? - correct answerbetween age 12-13 What is Auspitz sign? - correct answerthe appearance of small bleeding points after successive layers of scale have been removed from the surface of psoriatic papules or plaques APEA Predictor with complete answers latest updated 2024 What is the main difference between Kawasaki disease and scarlet fever? - correct answerKD has conjunctivitis and scarlet fever does not The rash associated with acute rheumatic fever - correct answererythema marginatum The rash associated with parvovirus aka slapped cheek aka fifth disease - correct answererythema infectiosum What does erythema marginatum of rheumatic fever look like? - correct answererythematous with a central clearing, not itchy The medical term for tennis elbow - correct answerlateral epicondylitis What is the leading cause of death in anorexia? - correct answersuicide Erikson stage from birth to 18 months - correct answertrust vs mistrust Erikson stage from 18 months to 3 years - correct answerautonomy vs shame and doubt Erikson stage from 3-5 years - correct answerinitiative vs guilt Erikson stage from 5-13 years - correct answerindustry vs inferiority Erikson stage from 13-21 years - correct answeridentity vs role confusion Erikson stage from 21-39 years - correct answerintimacy vs isolation Erikson stage from 40-65 years - correct answergenerativity vs stagnation Erikson stage for 65 and older - correct answerego integrity vs despair Macrobid/nitrofurantoin is contraindicated in which patients? - correct answerthose with renal insufficiency Treatment options for complicated UTI - correct answercipro or levo x 5-7 days Treatment options for uncomplicated UTI - correct answerbactrim x 3 days, macrobid x 5 days, fosfamycin 1g one time dose Olecranon bursitis is pain in which region of the elbow? - correct answerthe posterior elbow; will also have swelling and warmth Lateral epicondylitis manifests as what type of elbow pain? - correct answerweak grip and pain with gripping, pain with rotation of the wrist and forearm APEA Predictor with complete answers latest updated 2024 The medical term for golfer's elbow - correct answermedial epicondylitis How does the pain in lateral vs. medial epicondylitis differ? - correct answerlateral epicondylitis is worse with supination of the hand and medial epicondylitis is worse with pronation of the hand involves tapping a finger along the problem nerve - correct answertinel sign asking the patient to place both elbows on a table while keeping both forearms vertical and flexing both wrists - correct answerphalen sign Which nerve runs on the side of the thumb - correct answermedian nerve Which nerve runs on the side of the pinky finger - correct answerulnar nerve What is ankylosing spondylitis? - correct answeran inflammatory condition affecting the spine causing chronic back pain and stiffness that improves with exercise and enthesitis What is the treatment for ankylosing spondylitis? - correct answerrefer to rheumatology for possible DMARDs What are the s/s of polymyalgia rheumatica? - correct answerchronic pain in the proximal joints (shoulders and hips) bilaterally with stiffness lasting for more than 45 minutes What is the recommended treatment for polymyalgia rheumatica? - correct answerPO steroids and refer to Rheumatology to confirm the diagnosis Where are the biggest food sources of vitamin B12? - correct answerfish, meat, poultry, eggs, and dairy products, also fortified breakfast cereals and fortified nutritional yeasts What is the gold standard for the diagnosis of herpes? - correct answernucleic acid amplification test Most cases of acute sinusitis are viral and how can we treat them? - correct answertreat the symptoms with intranasal steroids and saline rinses What is the difference between chronic vs gestational hypertension? - correct answerchronic htn is present before 20 weeks gestation or 12 weeks after delivery; gestational hypertension is after 20 weeks gestation Which is the recommended diagnostic when a patient has a breast lump? - correct answerif less than 45, do an ultrasound. if older than 45, do a diagnostic mammogram. APEA Predictor with complete answers latest updated 2024 An overactive bladder is what type of incontinence? - correct answerurge incontinence What is the recommended treatment for urge incontinence? - correct answeranticholinergics like oxybutynin What is the recommended treatment for stress incontinence? - correct answeralpha agonists like sudafed, TCAs What is the recommended treatment for BPH? - correct answeralpha blockers like terazosin, doxazosin, tamsulosin Alcohol and marijuana worsen which type of incontinence? - correct answerurge Which SSRI is safest in pregnancy? Which poses the most risk? - correct answersertraline; paroxetine Infants born to women who smoke are at increased risk for what? - correct answerlow birth weight and preterm delivery What is the best marker of an acute MI? - correct answertroponin When do troponins peak after a MI? - correct answer12 hours When should an infant be able to display a conjugate gaze (track you all the way across the room)? - correct answer6 months Which medication is approved in Peds for the prevention of febrile seizures? - correct answerdiazepam Folic acid is which vitamin? - correct answervitamin B9 What is the dosage preferred for folic acid deficiency? - correct answer1-5mg daily Dosage of folic acid in pregnancy to prevent neural tube defects - correct answer400mcg daily What are ECG changes you may see on a patient with hypokalemia? - correct answertachyarrhythmas, ST depression, prolonged QT, and U wave present What are ECG changes you may see on a patient with hyperkalemia? - correct answerpeaked T waves A U wave present on EKG is thought to represent what? - correct answerrepolarization of the purkinje fibers APEA Predictor with complete answers latest updated 2024 How is permethrin applied to the skin for scabies? - correct answera one time application to all skin below the neck and then left on for 8-12 hours is sufficient After exposure to HIV, when will the patient seroconvert and become positive? - correct answer3 months Is age less than 18 years a risk factor for an ectopic pregnancy? - correct answerno; but personal history of PID, an IUD, previous ectopic pregnancy, previous tubal surgery, pathology, or ligation all are risk factors What is Tanner stage 5 of breast development in females? - correct answerprojection of the nipple only What are two common classes of drugs that interact with grapefruit juice? - correct answerstatins and CCBs Ankle inversion is a common complaint from a patient with what? - correct answera lateral ankle sprain How can we best diagnose coarctation of the aorta? - correct answerby comparing the upper and lower extremity blood pressures What is the most important risk factor for developing breast cancer in women of average risk? - correct answerage hey - correct answerstopped at top of page 7 What is the recommended treatment for blepharitis? - correct answererythromycin 1cm ribbon 5x daily for 7 days What is the recommended treatment for severe anorexia with delusions? - correct answer2nd atypical antipsychotics What lab values need to be monitored in a patient on Tizanidine? - correct answerLFTs and BP at baseline and 1 month after the max dose is achieved (can cause hepatotoxicity and hypotension) What class of drug is dabigatran (pradaxa)? - correct answerdirect thrombin inhibitor The most effective treatments for acne related to increased sebum production are what? - correct answeroral contraceptives and isotretinoin Oral contraceptive regimen in which both the estrogen and progesterone concentrations vary throughout the cycle - correct answerquadriphasic APEA Predictor with complete answers latest updated 2024 Oral contraceptive regimen in which the estrogen and progestin doses are fixed throughout the cycle - correct answermonophasic Oral contraceptive regimen in which in the estrogen dose remains the same for the first 21 days of the cycle, while progestin is lower in the first half and higher in the second half - correct answerbiphasic Oral contraceptive regimen in which the estrogen remains consistent and the progestin dose varies - correct answertriphasic Patients who are receiving beta blockers may find what treatment for allergies ineffective? - correct answertreatments containing epinephrine For a patient on warfarin who needs to receive a tetracycline antibiotic, how will we adjust their warfarin? - correct answertetracyclines depress prothrombin activity, same as warfarin, so we need to decrease the warfarin to avoid an increase in INR and significant bleeding What is the herbal, valerian root often used for? - correct answerinsomnia/sleep, ADHD, anxiety, depression What is the herbal, saw palmetto, often used for? - correct answerBPH What is the herbal, black kohosh, often used for? - correct answersymptoms of menopause - hot flashes What is the herbal, St. John's wart, often used for? - correct answerdepression What are the intrauterine transplacental infections that can be passed from mother to infant? - correct answerthink TORCH; toxoplasmosis, Other (syphilis, varicella zoster, parvovirus) Rubella, Cytomegalovirus and Herpes simplex, as well as tuberculosis. Note that chlamydia is NOT transplacental, but rather transvaginal during childbirth The antibiotic of choice for a dog bite - correct answeraugmentin; doxy if there is a penicillin allergy Obese women should gain how much weight during pregnancy? - correct answer11-20 pounds Overweight women should gain how much weight during pregnancy? - correct answer15-25 pounds Normal weight people should gain how much weight during pregnancy? - correct answer25-35 pounds APEA Predictor with complete answers latest updated 2024 Underweight people should gain how much weight during pregnancy? - correct answer28-40 pounds Pregnant women should increase their daily caloric intake by how many calories per day? - correct answer300 Immunizations are considered to be what level of prevention? - correct answerprimary Children can start to ride a tricycle around age _____ - correct answer3 A gray or white ring present around the peripheral iris - correct answerarcus senilis What causes arcus senilis? - correct answerit is a normal, benign condition with aging, but can also occur in younger people as a result of high cholesterol and triglycerides How is a diagnosis of neuroblastoma confirmed? - correct answervia serum catecholamine testing and a biopsy Are saturated or unsaturated fatty acids better for you? - correct answerunsaturated Which patient populations are thalassemias most common in? - correct answermediterranean countries (greece and turkey), asia, africa Is Meniere's disease most often unilateral or bilateral? - correct answerunilateral What type of hearing loss occurs in Meniere's disease? - correct answersensorineural Dix Hallpike maneuver is the standard clinical test for _______ - correct answerBPPV How do we test cerebellar function? - correct answerrapid alternating movements, finger to nose test, balance (romberg), gait The first sign of puberty in girls - correct answerthelarche (breast budding) When the uterine lining grows in to the uterine muscle wall - correct answeradenomyosis; this is often asymptomatic and may result in heavier periods What are the 4 components of Tetralogy of Fallot? - correct answerVSD, pulmonic stenosis, overriding aorta, right ventricular hypertrophy What does the murmur of Tet of Fallot sound like? - correct answerharsh systolic murmur at the left upper sternal border What is a tet spell? - correct answerwhen the child turns blue during crying or feeding APEA Predictor with complete answers latest updated 2024 How is lichen sclerosus treated? - correct answertopical steroids, topical calcineuronin inhibitors to suppress the immune system, topical or oral retinoids, phototherapy What is the tell-tale sign of malignant hypertension? - correct answerretinopathy and blurred vision; characterized by a diastolic BP of greater than 130 What is the difference between a Parkinsonian tremor and an essential tremor? - correct answera Parkinson tremor is also known as a resting tremor and is seen at rest, may look like pill-rolling. an essential tremor is bilateral, symmetric and seen when the hands are doing something What can worsen and improve an essential tremor? - correct answerworsened by stress or fatigue, improved by alcohol What is first-line treatment for trigeminal neuralgia? - correct answercarbamazepine (tegretol) or amtriptyline Are Parkinsonian tremors unilateral or bilateral at first? - correct answerunilateral Where is the TMJ best palpated? - correct answeranterior to the tragus What are the primary bacteria in dog bites? - correct answerstaph, strep, and pasteurella What is the gold standard treatment of giant cell arteritis? - correct answeroral steroids and potential artery biopsy by rheum Who is a candidate for Truvada for Prep? - correct answerif you are an IV drug user, have had a sexual partner with HIV in the last 6 months, do not consistently use a condom and engage in anal or vaginal sex, or have been diagnosed with a STD in the last 6 months Post exposure prophylaxis for someone with HIV should be completed when? - correct answerwithin 72 hours of the exposure Who should be screened for HIV? - correct answereveryone at least once from age 13- 64 if low risk, and then more frequently if high risk Should patients undergo gonorrhea testing to confirm a cure after receiving treatment? - correct answeronly if they had pharyngeal gonorrhea, not vaginal or anal Tanner stage 1 female breast developmental - correct answerprepubertal APEA Predictor with complete answers latest updated 2024 Tanner stage 2 female breast development - correct answerthelarche; the breasts bud as a small mound and the areola increases in diameter Tanner stage 3 female breast development - correct answerthe areola, breast, and nipple grow together in one mount Tanner stage 4 female breast development - correct answerthe areola and nipple separate from the breast to form a secondary mound Tanner stage 5 female breast development - correct answermature nipples project only Tanner stage 1 male genital development - correct answerprepubertal Tanner stage 2 male genital development - correct answertesticles and scrotum enlarge Tanner stage 3 male genital development - correct answertesticles continue to enlarge and the penis lengthens Tanner stage 4 male genital development - correct answerpenis lengthens, testicles enlarge, and the scrotum darkens Tanner stage 5 male genital development - correct answermature Antibiotic of choice for lyme disease - correct answerdoxycycline Antibiotic of choice for cellulitis - correct answerkeflex or cefdinir Antibiotic of choice for chlamydia - correct answerazithromycin Antibiotic of choice for syphilis - correct answerpenicillin G Antibiotic of choice for bacterial vaginosis - correct answerflagyl Antibiotic of choice for trichomoniasis - correct answerflagyl Antibiotic of choice for acute prostatitis - correct answercipro (or ceftriaxone if STI related) Antibiotic of choice for PID - correct answerceftriaxone and doxycycline Antibiotic of choice for pyelonephritis - correct answercephalexin or bactrim Antibiotic of choice for MRSA - correct answerbactrim, clindamycin, doxycycline Antibiotic of choice for dog or cat bite - correct answeraugmentin APEA Predictor with complete answers latest updated 2024 Antibiotic of choice for mastitis - correct answerkeflex Antibiotic of choice for chronic bacterial prostatitis - correct answerbactrim plus cipro or levo What is the earliest sign of diabetic nephropathy on labwork? - correct answermicroalbuminuria in the urine Recommended treatment for verruca (warts) - correct answertopical salicylic acid daily x 12 weeks Perineal pain is indicative of what in males? - correct answerprostatitis Gestational diabetes screening occurs between what weeks gestation? - correct answer24-48 When you ask the patient to take a deep breath in and then palpate over the RUQ abdomen. Positive when pain is elicited. This is a sign of ___________. - correct answerMurphy's sign; cholecystitis What are names of LABA medications? - correct answersalmeterol What is encopresis in children? - correct answerstool incontinence after the age of being toilet trained The medical term for being cross-eyed - correct answerstrabismus What is atopic dermatitis? - correct answereczema Tests used to diagnose developmental hip dysplasia - correct answerBarlow and Ortolani maneuvers What is the female athlete triad? - correct answerdisordered eating, amenorrhea, osteoporosis How do patients with aortic regurgitation typically present? - correct answerthey will have a long asymptomatic period followed by exercise intolerance, then dyspnea at rest. The left ventricle eventually fails unless the aortic valve is replaced. What does the rash of fifth disease (a viral exanthem) look like? - correct answermaculopapular rash that blanches easily and may be accompanied by joint aches APEA Predictor with complete answers latest updated 2024 What is the leading cause of CKD and ESRD in the united states? - correct answerdiabetes What is the usual recommendation about administering MMR and varicella immunizations? - correct answerthey should be given on the same day or at least one month apart What are the classic symptoms of an atopic pregnancy? - correct answeramenorrhea, vaginal bleeding, abdominal pain What is sideroblastic anemia? - correct answerwhen the bone marrow is unable to incorporate iron into hemoglobin. This may be either inherited or acquired. Can a pregnant woman get the MMR vaccine while pregnant? - correct answeryes Why are the carotid arteries auscultated for bruits? - correct answerthis is indicative of generalized atherosclerosis What is roseola? - correct answera viral exanthem characterized by a high fever for 3 days then abrupt stopping of the fever and a maculopapular rash that blanches. also known as exanthem subitum. What are signs indicative of a meningitis infection? - correct answerKernig and Brudzinski's signs Cancer in the bone marrow that results in skeletal destruction - correct answermultiple myeloma What is the average age of diagnosis of multiple myeloma? - correct answer65 a skin disorder that causes the nose to enlarge and become red, bumpy, and bulbous - correct answerrhinophyma What are physical exam findings common in alcohol abuse? - correct answertremors, macrocytosis (due to vitamin B12 and folate deficiencies common in alcoholics), hypertension, rhinophyma, peripheral neuropathy, teleangectasia, hepatosplenomegaly What is the first sign of sexual maturation in a male? - correct answeran increase in testicular size (tanner stage 2) The inability to name a familiar object - correct answeranomia What happens to a womans vital signs when she is in her second trimester of pregnancy? - correct answerher blood pressure decreases APEA Predictor with complete answers latest updated 2024 What is the most common cause of acute pharyngitis in children? - correct answerrespiratory viruses A depressed patient is started on an SSRI. When should another antidepressant be tried if there is no response? - correct answer8-12 weeks If the number of bacteria on a urinalysis are greater than ____________________, this is indicative of a UTI. - correct answer100,000 What are signs of a ruptured TM other than the obviously ruptured TM? - correct answermuffled hearing and bright red blood in the canal Osgood Schlatter disease produces pain where? - correct answerin the knees Growing pains usually occur when? - correct answerat nighttime Children with complaints of midbone pain should be worked for what MNM differential? - correct answeracute lymphocytic leukemia NSAIDs should be used in caution with what patient population? - correct answeranyone with hypertension because they cause sodium retention and increase blood pressure What is the most common cause of secondary dysmenorrhea? - correct answerendometriosis; symptoms include spotting in between periods, cervical stenosis or displacement, nodular or tender uterosacral ligaments When should AAA screening be performed via US? - correct answeronce for all males age 65-75 who have ever smoked What is responsible for the transmission of poison ivy? - correct answerthe oils from the plant itself. So the only way someone can pass it on to another person is if they still have oil from the plant on their skin (like if they haven't bathed). Note that the fluid inside the blisters is not contagious Actinic keratosis is a precursor for what? - correct answersquamos cell carcinoma What is papilledema? - correct answerswelling of the optic nerve and disc secondary to increased ICP. Often co-occurring with a headache Is sexual activity safe during pregnancy? - correct answerit can expose someone to STDs and it can also stimulate preterm labor due to stimulation of the lower uterine segment and release of oxtocin APEA Predictor with complete answers latest updated 2024 A report of "knee locking" is a classic complaint of what? - correct answera meniscal tear Can a pregnant woman be on levothyroxine during pregnancy? - correct answeryes, she should continue it and have TSH checked monthly because she may need a higher dose as thyroid hormone needs increase during pregnancy What characterizes mild persistent asthma? - correct answerasthma where symptoms occur more than twice weekly People who do not have a spleen (asplenic) are at a greater risk of what? - correct answermore susceptible to bacterial infections What 2 things put someone at the highest risk of developing pancreatitis? - correct answergallstones and alcohol abuse. Hypertriglyceridemia can also cause it. What are the RF for GI bleeding? - correct answerage 75 or older, warfarin use, daily NSAID use, chronic steroid use, PMHx of GI bleed When two or more RF for a GI bleed are present, we should not start a patient on aspirin without what? - correct answera daily PPI What medication is first line for the treatment of asymptomatic bacteriuria or UTI in pregnancy? - correct answernitrofurantoin Athletic amenorrhea increases one's risk of developing what? - correct answerosteoporosis; this is because there is a loss of body fat and body fat is what produces estrogen so estrogen levels decline An intervention being performed to prevent the occurrence of the same event happening again is an example of ___________ prevention. Give an example. - correct answersecondary; taking a daily aspirin to prevent another stroke after already having one What is Strattera (atomoxetine) used for? - correct answerADHD; side effect includes increased BP and HR so monitor closely What is the drug Danazol and what is it used for? - correct answeran androgen used in the treatment of endometriosis and fibrocystic breast disease Danazol, an androgen, has what potential side effects? - correct answeredema, hirsutism, hair loss, vocal changes The Mirena (Levonorgestrol) IUD stays in for how long? - correct answer5 years APEA Predictor with complete answers latest updated 2024 The flow of air coming in and out of the lungs in one inhalation and one exhalation - correct answerpeak expiratory flow What does an incentive spirometer measure? - correct answerpeak expiratory flow What happens in the luteal phase of the menstrual cycle? - correct answerLH and FSH decrease, the ruptured follicle closes after an egg has been released and forms the corpus luteum, which produces progesterone. Estrogen and progesterone remain high and it is those two things that cause the lining of the uterus to thicken further in preparation for fertilization What is a sign of the initial appearance of measurable histamines in a patient with allergic rhinitis? - correct answersneezing The bacteria responsible for syphilis is called what? - correct answertreponema pallidum The STI, chancroid, is caused by what responsible bacteria? - correct answerhaemophilus ducreyi What is the most common cause of septic arthritis? - correct answerstaph aureus The STI, mycoplasma genitalium, is unique in what way? - correct answerit lacks a cell wall and has a terminal tip-like structure which allows it to attach to and enter other cells. It then modulates the host's immune response What are the 3 secondary causes of type 2 diabetes? - correct answerprolonged physical or emotional stress, pregnancy, drugs such as steroids and hormonal contraceptives How does hyperinsulinemia in PCOS work? - correct answerinsulin resistance occurs in all places except for the ovary which remains sensitive to insulin. This insulin upregulates androgen production. Which drug classes are responsible for causing urinary retention? - correct answerantihistamines, TCAs, antispasmodics, bronchodilators, antipsychotics, carbidopa levodopa A 75 year old patient is experiencing progressive confusion and memory deterioration. Which condition mimics dementia that this patient should be tested for? - correct answerneurosyphilis Which hormones in late pregnancy can block insulin and cause gestational diabetes? - correct answercortisol, estrogen, and placental lactogen APEA Predictor with complete answers latest updated 2024 Target shaped skin lesions as a result of a hypersensitivity reaction to certain infections and drugs - correct answererythema multiforme An abnormal growth of squamos epithelium in the middle ear and mastoid - correct answercholesteatoma The cremasteric reflex is assessed in infants for what purpose? - correct answerto assess for retractile testicles; this occurs when the reflex is too strong and it pulls the testicle out of the body and into the groin; this is normal and should go away by puberty Patient populations who may present atypically when having a MI include who? - correct answerdiabetics, females, elderly A disorder that results in the destruction of platelets due to an autoimmune response - correct answeridiopathic thrombocytopenia purpura Pelvic inflammatory disease is most commonly caused by what? - correct answerchlamydia and gonorrhea A protruding abdominal mass after bending over, lifting, or coughing, and is painful with coughing, is often what in adults? - correct answeringuinal hernia Hiatal hernias cause what type of symptoms? - correct answerGERD What is the name of an immature female gamete? - correct answeroocyte The fused ovum and sperm once fertilization has occurred - correct answerzygote What are conditions that are often co-occurring with alopecia areata? - correct answeratopic dermatitis, vitiligo, thyroid disease, collagen-vascular diseases, Down syndrome The medical term for eczema - correct answeratopic dermatitis The leading cause of death in infants under the age of 1 - correct answersudden unexplained infant death How is varicella zoster transmitted? - correct answervia the respiratory tract The presence of pruritic papules, macules, and vesicles in various stages of healing - correct answervaricella zoster What is presbycusis? - correct answerage related hearing loss APEA Predictor with complete answers latest updated 2024 Age is the most common cause of presbycusis, but what other conditions may play a role in quickening the onset of this? - correct answerwhite, infections, smoking, hypertenson, diabetes, vascular disease, immune disorders, hormonal factors The fascia layer of the male anatomy that the anterior 2/3s of the testicle resides in - correct answertunica vaginalis A subtype of strabismus that describes an inward turning of the eye - correct answeresotropia The abducens nerve (CN VI) is responsible for what? - correct answerlateral movement of the eye outward The trochlear nerve is responsible for what extraocular movement? - correct answerdown and inward movement Why is primary open-angle glaucoma considered a chronic glaucoma? - correct answerthere is a SLOW rise in intraocular pressure A benign epidermal tumor that is a well-demarcated, coin-like, pigmented lesion with a stuck on appearance - correct answerseborrheic keratosis The most common precancerous skin lesion that results from years of repeated sun exposure - correct answeractinic keratosis What happens during the follicular phase of the menstrual cycle? - correct answerFSH and LH increase which stimulate the production of estrogen. Ultimately, it is estrogen that prepares the uterus for ovum fertilization A superficial dermatophyte/fungal infection of the skin of the scalp, eyebrows, and eyelashes - correct answertinea capitis Bacterial invasion of the renal parenchyma - correct answerpyelonephritis What is phimosis vs. paraphimosis? - correct answerphimosis is tightening of the foreskin over the glans that prevents retraction; paraphimosis is when you can retract the foreskin but you can't pull it back up What is a common characteristic of a person with Graves disease? - correct answerophthalmopathy causing lymphocytes to infiltrate the extraocular muscle, resulting in paralysis of EOMs, involvement of the optic nerve, and corneal ulcers Which phase of the menstrual cycle begins with a surge in LH and FSH? - correct answerovulatory phase APEA Predictor with complete answers latest updated 2024 What are the 5 primary structures of the immune system? - correct answerlymph nodes, thymus, tonsils, spleen, bone marrow Bacteria most commonly responsible for pyelonephritis - correct answere coli What causes back pain in pregnancy? - correct answercurvature of the lumbosacral vertebrae What is dacrocystitis? - correct answerinflammation of the nasolacrimal duct due to duct obstruction or infection What are potential causes of blepharitis? - correct answerbacterial infection, dry eyes, rosacea What are s/s of croup? - correct answerhoarseness, cough, inspiratory stridor, possible respiratory distress due to narrowing of the larynx Women who become pregnant later in life have an increased risk of what? - correct answerbreast cancer What are things women can do to decrease their risk of breast cancer? - correct answerregular physical activity, breastfeeding Does an increase or decrease in blood pH affect the rate and degree of sickling in sickle cell anemia? - correct answera decrease in blood pH increases the rate What is the difference between Prinzmetals and stable (classic angina)? - correct answerPrinzmetals may occur randomly and while at rest, whereas stable angina is usually only with physical activity Why may infection of the palatine tonsils become serious very quickly? - correct answerthe area is composed of loose connective tissue What are cutaneous xanthomas? - correct answerlocalized lipid deposits caused by elevated lipids or LDL Uterine prolpase is caused by stretching of the uterine ____________ - correct answerligaments What causes bacterial vaginosis? - correct answera decrease in lactobacillus species within the vagina Where is acanthosis nigricans most likely to occur? - correct answeraxillae, neck, groin What causes subjective tinnitus? - correct answerabnormal neuronal activity APEA Predictor with complete answers latest updated 2024 HIV produces cellular immunity deficiency by depleting what? - correct answerhelper T cells What is the function of B cells in the immune system? - correct answerdevelopment of antibodies When does the male fetus begin to produce testosterone and how? - correct answeraround 9 weeks gestation due to the Leydig cells What is the pathophys of Addison's disease? - correct answercauses decreased or absent aldosterone levels resulting in increased excretion of sodium and water by the kidneys, and hyperkalemia What is the most serious complication of Addison's disease? - correct answeracute adrenal insufficiency or adrenal crisis What are the 4 heart defects of Tetralogy of Fallot? - correct answerVSD, right ventricular hypertrophy, overriding aorta, pulmonary stenosis What are the s/s of Kawasaki disease? - correct answerthink CREAM; conjunctivitis, rash, erythema of hands and soles of feet, adenopathy (cervical), mucous membranes (strawberry tongue) A palpable softening of the cervical isthmus during pregnancy - correct answerHegar's sign A bluish tint to the vagina and cervix due to increased vascularity during pregnancy - correct answerChadwick's sign What is Paget's disease? - correct answerwhere there is abnormal bone destruction or growth of bone. It is also a malignant disease of the breast. Does Meniere's disease result in hearing loss? - correct answeryes, gradual but progressive sensorineural hearing loss What is another term for chronic bladder pain syndrome? - correct answerinterstitial cystitis What classifies someone as having "prediabetes"? - correct answerhgba1c between 5.7 and 6.4 What classifies someone as having diabetes? - correct answerhgba1c 6.5 or greater APEA Predictor with complete answers latest updated 2024 A reduction in vision in one or both eyes causing loss of binocular vision - correct answeramblyopia What is Brudzinski's sign? - correct answerthe patient lies supine and the NP flexes the patients neck forward. If both hips and knees flex at the same time, this is indicative of meningitis A child presents with erythematous papules and vesicles, that are weeping, oozing, and crusty. They are located all over the forehead, wrists, elbows, and backs of knees. What is this most likely to be? - correct answeratopic dermatitis In older adults, what is an audible S3 sound usually a sign of? - correct answerleft ventricular hypertrophy from heart failure, or cardiomyopathy How is cervical radiculopathy different from cervical myelopathy? - correct answerin radiculopathy, there is nerve root compression that results in burning pain and n/t in the neck and one arm. In myelopathy, there is pain with bilateral weakness and n/t in both the upper and lower extremities Other than scratching the medial aspect of the thigh, how else can we elicit the cremasteric reflex in a child? - correct answerhave them sit cross legged and blow into a balloon. This will increase intraabdominal pressure and stimulate the reflex. What is the Lachman's test and what does it identify? - correct answerused to assess for ACL tear; the knee is held passively at 30 degrees flexed and then one hand stabilizes the femur while the other applies a gentle anterior force to the proximal tibia. If it moves beyond it's normal ROM, there may be an ACL tear When should the testicles fully descend into the scrotum by in an infant? - correct answer6 months of age. If not descended by this time, then refer to Urology at 6 months although surgical correction (orchipexy) may not be performed until 1 year What is another name for the moro reflex? - correct answerthe startle reflex; both arms should extend outwards when this is positive If a moro reflex in an infant is only positive in one arm, then what do you need to consider? - correct answerexamine the clavicle for a potential fracture Which model is used to designate risk factors for melanoma? - correct answerthink HARMM; history, age, regular dermatologist absent, mole changing, male gender What is a sign of an ankle fracture on physical exam? - correct answerpoint tenderness over the posterior malleolus with an inability to bear weight after 4 or more steps APEA Predictor with complete answers latest updated 2024 Test of thumb movement by instructing the patient to place his thumb in the palm and then move the wrist toward the midline in ulnar deviation - correct answerFinkelsteins test What does the murmur associated with mitral regurg sound like? - correct answerpansystolic, harsh murmur heard loudest at the apex with radiation toward the left axilla When auscultating the apex of the heart in a child older than 7, the bell of the stethoscope should be placed where? - correct answerfifth intercostal space to the left of the midclavicular line; if younger than 7, it is best heard at the third or fourth intercostal space What is the main sign of Kaposi's sarcoma? - correct answerraised or flat deep purple lesions in the mouth Where do lesions of Kaposi's sarcoma usually manifest? - correct answerin the mouth, GI tract, or lungs A patient is experiencing a dull achy pain in the epigastric area with eating. This type of pain is consistent with what? - correct answera gastric ulcer How is the pain of a gastric ulcer vs. a duodenal ulcer different? - correct answerthe pain is worsened with eating with a gastric ulcer but improved with eating in a duodenal ulcer What are s/s of glaucoma? - correct answerloss of peripheral vision, halos around lights, difficulty focusing on near objects Higher level functioning, such as using the telephone, doing laundry, taking medicine, etc. are considered to be called what? - correct answerinstrumental activities of daily living An inability to extend the knee is a sign of what? - correct answera partial or complete patellar tendon tear Which part of the spine controls the ankle DTRs? - correct answersacral nerve 1 Which part of the spine controls the knee DTRs? - correct answerlumbar nerves 2, 3, 4 Which part of the spine controls the biceps DTRs? - correct answercervical nerves 5 and 6 APEA Predictor with complete answers latest updated 2024 Which part of the spine controls the triceps DTRs? - correct answercervical nerves 6 and 7 What are s/s of carpal tunnel syndrome? - correct answerdropping objects, unable to twist lids off of jars, aching at the wrist or forearm, numbness of the first 3 digits of the hand Which movement indicates a nuchal rigidity sign when working a patient up for meningitis? - correct answerneck stiffness and resistance when flexing the neck forward What is Kernig's sign? - correct answerflex the patients leg at both the hip and the knee, then straighten the knee. Pain and resistance to extension of the knee are positive for meningitis. A patient who is unable to touch his left scapula with his right hand likely has what? - correct answera rotator cuff tear A widened pulse pressure greater than or equal to 60 in an older adult is a risk factor for what? - correct answercardiovascular disease, stroke, renal disease The ethmoidal sinuses are located where? - correct answerbetween the eyes Where do the sphenoid sinuses lie? - correct answerjust behind the ethmoid sinuses Where do the maxillary sinuses lie? - correct answeraround the nasal cavity What describes a stage III pressure ulcer? - correct answerfull thickness tissue loss and subcutanous fat may be present, but bone, tendon, or muscle are not exposed Dislocation of the elbow joint caused by the sudden pull on the extended pronated forearm - correct answernursemaid's elbow Localized tenderness over the ankle joint is a sign of what? - correct answera ligamentous injury; point tenderness and inability to bear weight is more of a sign of an ankle fracture Describe renal adaptation of the newborn - correct answerthe kidneys are structurally complete but physiologically immature. the infant is at increased risk of metabolic acidosis due to the kidneys inability to concentrate urine and adapt to fluid and electrolyte stress Where is the best place to locate the twelfth rib? - correct answerbetween the spine and lateral chest APEA Predictor with complete answers latest updated 2024 What is biliary colic? - correct answerpain in the epigastric region or RUQ that radiates to the right scapula or shoulder; indicative of cholecystitis Costochondritis occurs when there is inflammation of the cartilage where? - correct answerthat connects the ribs to the sternum What aggravates costochondritis? - correct answermovement of the chest, trunk, and arms Which segments of the spine control the plantar reflexes? - correct answerlumbar 5 and sacral 1 A softening of the skin at the angles of the mouth which may be due to nutritional deficiency - correct answerangular cheilitis Angular cheilitis can be caused by what? - correct answernutritional deficiency What causes actinic cheilitis? - correct answerexcessive exposure to sunlight; most often affects the lower lip Intention tremors appear with movement and worsen when? - correct answeras the target gets closer Nausea, diaphoresis, and pallor triggered by a fearful or unpleasant event - correct answervasovagal syncope Which conditions are known to cause polyuria? - correct answerlithium toxicity, uncontrolled diabetes, disorders of the posterior pituitary and hypothalamus (hypogonadism), and hypokalemic nephropathy Breath sounds heard over the periphery of the lung fields are what? - correct answervesicular; they are quiet and wispy What do bronchovesicular breath sounds sound like? - correct answerfull inspiratory phase with a shortened and quieter expiratory phase Thenar atrophy suggests what? - correct answera median nerve disorder Does the ulnar nerve innervate the pinky or the thumb? - correct answerpinky The pregnant woman who has a BMI of less than 18.5 should gain how many pounds in pregnancy? - correct answer28-40 pounds The pregnant woman who has a BMI of 18.5-24.9 (normal) should gain how many pounds in pregnancy? - correct answer25-35 pounds APEA Predictor with complete answers latest updated 2024 Which of these manifestations should a nurse expect to observe in a 3-month-old infant who is diagnosed with dehydration? a. Hyperreflexia. b. Tachycardia. c. Bradypnea. d. Agitation. - correct answerb. Tachycardia. When assessing a client's risk of developing nosocomial infection, a nurse plans to determine potential entry portals, which include: a. the urinary meatus. b. vomitus. c. contaminated water. d. sexual intercourse. - correct answera. the urinary meatus. A client who is on the inpatient psychiatric unit has a history of violence. Which of these actions should a nurse take if the client is agitated? a. Encourage the client to verbalize feelings. b. Lock the client in a secluded room. c. Ask the other clients to give feedback regarding the client's behavior. d. Ignore the client's inappropriate behavior. - correct answera. Encourage the client to verbalize feelings. Which of these measures should a nurse include when planning care for a school-aged child during a sickle cell crisis episode? a. Monitoring for signs of bleeding. b. Providing pain relief. c. Administering cool sponge baths to reduce fevers. d. Offering a high calorie diet. - correct answerb. Providing pain relief. Which of these instructions should a nurse include in the plan of care for a 32-week gestation client who had an amniocentesis today? a. "Drink at least six glasses of fluids during the next six hours after the test." b. "Call the clinic if you experience any abdominal cramps." c. "Don't be concerned if you have some vaginal spotting in the next 12 hours." d. "When you get home, stay on bed-rest for the next 48 hours." - correct answerb. "Call the clinic if you experience any abdominal cramps." An adolescent has a nursing diagnosis of fatigue related to inadequate intake of iron- rich foods. Selection of which of these lunches by the client indicates a correct understanding of foods high in iron content? a. Peanut butter and jam sandwich. b. Chicken nuggets with rice. c. Tuna salad sandwich. d. Beefburger with cheese. - correct answerd. Beefburger with cheese. APEA Predictor with complete answers latest updated 2024 A client has been admitted with acute pancreatitis. Which of these laboratory test results supports this diagnosis? a. Elevated serum potassium level. b. Elevated serum amylase level. c. Elevated serum sodium level. d. Elevated serum creatinine level. - correct answerb. Elevated serum amylase level. Which of these manifestations, if assessed in a client who is two-hours postoperative after abdominal surgery, should a nurse report immediately? a. Vomiting and a pulse rate of 106/minute. b. Respiratory rate of 12/minute and urine dribbling. c. Blood pressure of 100/60 mm Hg and wound discomfort. d. Urine output of 100 mL/hr and flushed skin. - correct answera. Vomiting and a pulse rate of 106/minute. Which of these observations of a student nurse's behavior while interacting with a client who is crying indicates a correct understanding of therapeutic communication? a. The student maintains continuous eye contact with the client. b. The student places one arm around the client's shoulder? c. The student sits quietly next to the client. d. The student leaves the room to provide privacy for the client. - correct answerc. The student sits quietly next to the client. Which of these actions should a nurse take initially if a client who is diagnosed with diabetes mellitus develops tremors and ataxia? a. Measure the client's blood sugar level. b. Administer a concentrated form glucose to the client. c. Administer a prn dose of insulin. d. Measure the client's urine for ketones. - correct answera. Measure the client's blood sugar level. An elderly client is at increased risk of developing drug toxicity to prescribed medications due to declining hepatic and renal functioning. Which of these strategies should a nurse plan to decrease this risk? a. Increasing the time interval between medication doses. b. Limiting the client's oral fluid intake. c. Administering the medications with meals. d. Encouraging the client to void every three to four hours. - correct answera. Increasing the time interval between medication doses. A client has persistent paranoid delusions that the food on the unit is poisoned. Which of these measures should a nurse include in the client's care plan? a. Explaining that staff does not poison clients. b. Focusing on how the hospital staff helps clients. c. Allowing the client to eat food from sealed containers. APEA Predictor with complete answers latest updated 2024 d. Telling the client that not eating the food that is served will result in privilege restrictions. - correct answerc. Allowing the client to eat food from sealed containers. Thrombophlebitis is a complication that may result due to surgery. Which of these actions should a nurse take in the operating room to prevent this complication from occurring? a. Gatch the knee of the bed. b. Administer anticoagulants preoperatively. c. Apply sequential compression devices. d. Maintain the legs in a dependent position. - correct answerc. Apply sequential compression devices. When discussing weigh gain during pregnancy, a nurse should recommend that the total weight gain for a pregnant client who is at ideal body weight for her height is: a. at least 15 pounds. b. 15 to 20 pounds. c. 25 to 35 pounds. d. at least 45 pounds. - correct answerc. 25 to 35 pounds. Which of these manifestations, if reported by a client who is 10-weeks-pregnant, supports the diagnosis of ruptured tubal pregnancy. a. Sharp unilateral abdominal pain. b. Uncontrollable vomiting. c. Marked abdominal distention. d. Profuse vaginal bleeding. - correct answera. Sharp unilateral abdominal pain. Which of these assignments, if made by a nurse to a nursing assistant, indicates that the nurse needs additional instructions regarding the principles of delegation? a. "Please bathe the client in room 12, and then bring the client to the dining room for breakfast by 9 A.M." b. "Please bathe the client in room 10, administer a back rub, and then evaluate if the back rub eased the client's discomfort." c. "Please measure the intake and output for the client's in rooms 8. 9. and 10, and record each on the intake/output sheets by 2 P.M." d. "Please toilet the clients in rooms 11, 12, and 13 mid-morning and after lunch." - correct answerb. "Please bathe the client in room 10, administer a back rub, and then evaluate if the back rub eased the client's discomfort." A client has the following order for regular insulin (Humulin R) on a sliding scale: Blood sugar 150-180 mg: Give 2 units regular insulin Blood sugar 181-200 mg: Give 4 units regular insulin Blood sugar 201-220 mg: Give 6 units of regular insulin Blood sugar above 220 mg: Call MD APEA Predictor with complete answers latest updated 2024 Which of these instructions should be included in the teaching plan for the parents of a 10-month-old infant who is admitted to the hospital for failure to thrive? a. Advise the mother to make sure the infant drinks the entire bottle at each feeding. b. Encourage the mother to feed the infant slowly in a quiet environment. c. Teach the mother to position the infant on the abdomen following feedings. d. Instruct the mother to play actively with the infant during bottle feedings. - correct answerb. Encourage the mother to feed the infant slowly in a quiet environment. When a newborn is 48 hours old, a nurse notes that the child is jaundiced. The nurse should recognize which of these conditions as a probable cause of the newborn's jaundice? a. Dehydration. b. Liver immaturity. c. ABO incompatibility. d. Gallbladder immaturity. - correct answerb. Liver immaturity. Which of these items should a nurse removed from the food tray of a client who is on a sodium-restricted diet? a. Packet of a salt substitute. b. Grapefruit juice. c. Container of jelly. d. Ketchup. - correct answerd. Ketchup. Which of these statements, if made by a client who had a total hip replacement, would indicate a correct understanding of the postoperative instructions? a. "I will stoop carefully to pick up items from the floor." b. "I will use a raised toilet seat in the bathroom." c. "I will bend forward when tying my shoes." d. "I will put my leg through the full range of motion each day." - correct answerb. "I will use a raised toilet seat in the bathroom." Which of these measures should a nurse include when planning care for an 88-year-old client who is admitted to the hospital with pneumonia? a. Restricting visitors to the client's immediate family members. b. Limiting the client care activities to no more than five minutes each. c. Allowing the client to perform self-care as tolerated. d. Providing the client with a non-stimulating environment. - correct answerc. Allowing the client to perform self-care as tolerated. A client, who is newly diagnosed with cancer says to anurse, "I suppose I need to complete all unfinished business as soon as possible." Which of these responses is appropriate? a. "Yes, you should do this immediately. b. "Don't you think you should stay focused on your treatment for now? c. "Exactly what things are you talking about?" APEA Predictor with complete answers latest updated 2024 d. "It sounds like you are concerned with your diagnosis." - correct answerd. "It sounds like you are concerned with your diagnosis." Which of these interventions should plan for a child who is receiving chelation therapy for lead poisoning? a. Keeping an accurate record of intake and output. b. Instituting measures to prevent skeletal fractures. c. Maintaining isolation precautions. d. Maintaining strict bed rest. - correct answera. Keeping an accurate record of intake and output. A nurse obtains these vital signs on an adult client. Which finding should the nurse follow-up first? a. Heart rate, 60/minute and regular. b. Respiration, 30/minute and deep. c. Temperature, 97.1 °F (36.2 °C) d. Blood pressure, 136/86 mm Hg - correct answerb. Respiration, 30/minute and deep. When determining the duration of a uterine contraction, a nurse should measure the contraction from the: a. beginning of one contraction to the end of that contraction. b. end of one contraction to the beginning of the next contraction. c. beginning of one contraction to the beginning of the next contraction. d. strongest point of one contraction to the strongest point of the next contraction. - correct answera. beginning of one contraction to the end of that contraction. A nurse should recognize which of these signs is a probably sign of pregnancy? a. Frequency of urination. b. Positive pregnancy test. c. Nausea in the morning. d. Abdominal distention. - correct answerb. Positive pregnancy test. All of these clients are on bed rest. Which one is the most at risk to develop skin breakdown? a. An 82-year-old client who bathes once a week. b. An 83-year-old client who applies powder after drying the skin. c. An 84-year-old client who has been NPO for four days. d. An 85-year-old client who has coronary artery disease. - correct answerc. An 84- year-old client who has been NPO for four days. A client diagnosed with type 1 diabetes mellitus has a glycosylated hemoglobin A1c of 4.2%. A nurse should interpret this to mean that the client has: a. had a period of sustained hyperglycemia. b. been non-compliant with home management. c. been in relatively good diabetic control. APEA Predictor with complete answers latest updated 2024 d. eaten a high carbohydrate snack just prior to testing. - correct answerc. been in relatively good diabetic control. A nurse is caring for a client with burns and in reverse isolation. Which measures should the nurse include? a. Wearing disposable gloves when chaging the dressings. b. Having the client wear goggles when staff is in the room. c. Wearing a gown, mask, and gloves when providing care to the client. d. Disposing of the client's soiled laundry in a red bag. - correct answerc. Wearing a gown, mask, and gloves when providing care to the client. A physician has ordered 100 mg of Amoxicillin po for a child. The available liquid amoxicillin is 250 mg/5 mL. How many milliliters should a nurse administer? a. 1.0 b. 1.5. c. 2.0 d. 2.5 - correct answerc. 2.0 A nurse charts on all assigned clients at 2:00 P.M. The nurse then remembers something that happened at 9:00 A.M. to a client who was not charted. Which of these actions should the nurse take? a. Include the 9:00 A.M. scenario in the shift report. b. Enter the scenario after the original 2:00 P.M. charting and mark it as a "late entry". c. Put the information in the margin and indicate the accurate time placement by drawing an arrow. d. Draw a line through the previous charting with "error" and then re-record everything, including the new information. - correct answerb. Enter the scenario after the original 2:00 P.M. charting and mark it as a "late entry". While giving a bath to a client, a nurse notices that the client's back appear reddened. Which of these interpretations and additional assessments should the nurse make? a. The client's skin is sensitive to touch; lightly rub the client's chest area. b. The client has decreased circulation; palpate the peripheral pulses. c. The client is showing signs of pressure; press on the skin and observe for a return of color. d. The client is allergic to the soap; check the extremities for discoloration. - correct answerc. The client is showing signs of pressure; press on the skin and observe for a return of color. A newborn is placed under fluorescent light as part of the treatment for physiologic jaundice. During the duration of the newborn's treatment, a nurse should: a. cover the newborn's closed eyes with patches. b. measure the newborn's pulse and respirations every two hours. c. keep the newborn under the light at all times, even during the feedings. APEA Predictor with complete answers latest updated 2024 A young healthy adult, who has been exercising in hot weather, has fatigue, loss of appetite, and lightheadedness. Which of these assessments should a nurse make? a. Determine the client's preferred diet. b. Measure the client's body temperature. c. Auscultate the lungs. d. Ascertain the client's typical sleep pattern. - correct answerb. Measure the client's body temperature. Which of these nursing measures is the priority for a child who has hemophilia and who sustains a leg injury? a. Ensuring adequate hydration for the child. b. Soaking the child's injured leg in warm water. c. Administering the missing factor VIII to the child. d. Transfusing one unit of whole blood to the child. - correct answerc. Administering the missing factor VIII to the child. Which of these outcomes should a nurse focus on for a client who had a bronchoscopy two hours ago? a. Preventing hemorrhage. b. Preventing pneumonia. c. Preventing aspiration. d. Preventing dehydration. - correct answerc. Preventing aspiration. A client who had a coronary artery bypass graft four days ago suddenly develops sinus tachycardia and reports shortness of breath and dizziness. Which of these interpretations and actions should a nurse take? a. This is an expected occurrence following bypass surgery; continue to monitor the client. b. This indicates normalization of the blood pressure; hold all anti-hypertensive medications. c. This may be an early sign of heart failure; notify the physician. d. This indicates hypoxia; administer oxygen at 5/L per minute. - correct answerc. This may be an early sign of heart failure; notify the physician. Which of these lunch selections, if made by a client who has congestive heart failure, should a nurse recognize as indicative of a need for additional instructions? a. Cottage cheese with fresh fruit salad, whole wheat bread, and herbal tea. b. Baked chicken with brown rice, mixed green salad, and iced coffee. c. Egg salad sandwich with mayonnaise, pickles, and seltzer water. d. Beef tenderloin, carrots, mashed potatoes, and a baked apple. - correct answerc. Egg salad sandwich with mayonnaise, pickles, and seltzer water. Which of the statements if made by a client who is take furosemide (Lasix), supports a nursing diagnosis of knowledge deficit? APEA Predictor with complete answers latest updated 2024 a. "This medication will increase the amount and frequency of my urination." b. "This medication must be taken, even on days when I fell well." c. "I will need to add more salt to my diet because this medication will increase its excretion." d. "I should change my position slowly to avoid dizziness related to this medication." - correct answerc. "I will need to add more salt to my diet because this medication will increase its excretion." Which of these statements, if made by a client who has chronic obstructive pulmonary disease, indicates improvement? a. "I hope to attend my grandson's graduation next month." b. "I can now walk one more block than I could last month." c. "I take several quick breaths when I begin to cough." d. "I do my breathing exercises in the evening after I eat dinner." - correct answerb. "I can now walk one more block than I could last month." An 8-month-old infant is admitted to the hospital because of failure to thrive. Which of these actions should a nurse plan? a. Limit the parents' interactions with the infant. b. Consistently assign the care of the infant to the same staff. c. Rotate assignments so that all staff can evaluate the infant. d. Limit the infant's activity until the cause of the problem is identified. - correct answerb. Consistently assign the care of the infant to the same staff. Which of these actions should a nurse include to enhance the effectiveness of client teaching sessions? a. Include all content in one session so as not to overwhelm the client. b. Initially demonstrate and explain the procedure to the client. c. Avoid repetition of content. d. Include all clients on the unit in the sessions. - correct answerb. Initially demonstrate and explain the procedure to the client. Which of these laboratory test results is more important for a nurse to assess for a client who reports chest pain? a. WBC count. b. PTT level. c. Troponin level. d. Hemoglobin. - correct answerc. Troponin level. A nurse should explain to a primigravida that urine tests will be done at each prenatal visit throughout the pregnancy to measure: a. specific gravity and pregnancy hormones. b. culture and white blood cell count. c. glucose and protein. d. bacteria and red blood cell count. - correct answerc. glucose and protein. APEA Predictor with complete answers latest updated 2024 Which of these manifestations should a nurse expect to observe in a client who is diagnosed with paranoid schizophrenia? a. Regression. b. Suspiciousness. c. Catatonia. d. Hyperactivity. - correct answerb. Suspiciousness. Which of these measures should an emergency room nurse include when speaking with a family experiencing the loss of an infant from Sudden Infant Death Syndrome (SIDS)? a. Explaining to the parents how SIDS could have been predicted. b. Discouraging the parents from viewing the infant's body. c. Encouraging the parents to take the opportunity to say goodbye. d. Interviewing the parents in-depth about the circumstances of the infants death. - correct answerc. Encouraging the parents to take the opportunity to say goodbye. Which of these assessments is the priority for a client who is admitted with recurrent depression? a. Previous episodes of depression. b. Compliance with prescribed medications. c. Presence of a suicide plan. d. Problems with communication. - correct answerc. Presence of a suicide plan. Which of these changes in the assessment data of a child who has congestive heart failure should a nurse recognize as indicative of a therapeutic response to prescribed medication therapy? a. Increased weight. b. Increased urine output. c. Increased respiratory rate. d. Increased heart size. - correct answerb. Increased urine output. Which of these assignments, if delegated to unlicensed assistive personnel (UAP) by a nurse, is appropriate? a. The UAP is assigned to measure a client's intake and output. b. The UAP is assigned to assess a client's lung sounds. c. The UAP is assigned to teach a client about diet restrictions. d. The UAP is assigned to change a client's postoperative wound dressing. - correct answera. The UAP is assigned to measure a client's intake and output. A client who has a history of asthma develops an acute asthma attack. Which of these questions should a nurse ask when assessing the etiology of this attack? a. "Have you eaten any new foods recently?" b. "How many hours did you sleep last night?" c. "Are you exercising every day?" APEA Predictor with complete answers latest updated 2024 A nurse should assist a pregnant client who is in the first trimester to achieve the developmental task of this stage of pregnancy, which is: a. accepting the fact that she is pregnant. b. accepting the fact that the fetus is a separate being. c. accepting that she will soon deliver the child. d. accepting that her body image has changed. - correct answera. accepting the fact that she is pregnant. When interacting with a client who is paranoid, a nurse should: a. use touch to place the client at ease. b. maintain a caring facial expression. c. stand close to the client. d. maintain a professional attitude towards the client. - correct answerd. maintain a professional attitude towards the client. Which of these tasks is appropriate for a nurse to delegate to a nursing assistant in an acute care unit? a. Feeding a client who was admitted with a stroke yesterday. b. Ambulating a client who was admitted with a myocardial infarction yesterday. c. Measure the blood pressure of a client who was admitted with an asthma attack yesterday. d. Suctioning the tracheostomy that was performed on a client yesterday. - correct answerc. Measure the blood pressure of a client who was admitted with an asthma attack yesterday. Which of these techniques should a nurse plan to use with a client who is delusional? a. Explore the delusion so the client will know it is false. b. Explain clearly why the client's belief is incorrect. c. Focus on reality-based topics. d. Avoid speaking with the client when he/she is delusional. - correct answerc. Focus on reality-based topics. Which of the following manifestations should a nurse recognize as suggestive of right- sided heart failure? a. Cool extremities and frothy sputum. b. Jugular vein distention and pedal edema. c. Orthopnea and frequent cough at night. d. Weight loss and lower calf pains. - correct answerb. Jugular vein distention and pedal edema. Which of these statements, if made by a nursing student prior to a sterile dressing change, is correct? a. "I understand that if objects touch other objects on the sterile field they are considered contaminated." APEA Predictor with complete answers latest updated 2024 b. "I understand that sterile objects that are below my waist are considered contaminated." c. "I understand that all objects in the sterile field must be dry." d. "I understand that contaminated objects can be used if rinsed with an antimicrobial solution." - correct answerb. "I understand that sterile objects that are below my waist are considered contaminated." A nurse reviews a client's prenatal record and notes that the client's last menstrual period (LMP) was on September 18th. Using the Naegele's rule, the nurse should calculate that the client's expected date of delivery (EDD) will be: a. May 11th. b. May 25th. c. June 11th. d. June 25th. - correct answerd. June 25th. Which of these instructions should a nurse give to a client who has venous insufficiency regarding the use of elastic stockings (TEDs)? a. "Bunch the TEDs up and pull them on like socks." b. "Lower the TEDs to your ankles if your legs ache." c. "Keep the TEDs on at all times." d. "Put the TEDs on before you get up in the morning." - correct answerd. "Put the TEDs on before you get up in the morning." A nurse assesses a client who is scheduled for a total abdominal hysterectomy at 10:00 A.M. WHich of the factors should the nurse recognize as most likely to influence the outcome of the surgery? a. The client has voided two times since 5:00 A.M. b. The client is not able to demonstrate leg exercises because of osteoarthritis. c. The client takes one acetylsalicylic acid (baby Aspirin) daily. d. The client reports mouth dryness. - correct answerc. The client takes one acetysalicylic acid (baby Aspirin) daily. A client's urine output is 500 mL in 24 hours. Which of these actions should a nurse take? a. Report the findings to the physician. b. Obtain an order for a diuretic. c. Encourage the client to limit fluid intake. d. Record the finding and continue to monitor the client. - correct answera. Report the findings to the physician. A nurse should question an order for a potassium chloride intravenous infusion for which of these clients? a. A client who has hypoxia. b. A client who is obese. c. A client who has anuria. APEA Predictor with complete answers latest updated 2024 d. A client who is congested. - correct answerc. A client who has anuria. A 22-year-old college student has a heart rate that is 48/minute and regular during a routine physical examination. Which of these questions should a nurse consider when analyzing this heart rate? a. Is this student an athlete? b. Does this student smoke? c. How much alcohol does this student drink? d. Is this student feeling anxious? - correct answera. Is this student an athlete? Which of the following clients should a nurse recognize is most likely to develop diabetic ketoacidosis? a. A 23-year-old who has type 1 diabetes mellitus and is being treated for a tooth abscess. b. A 31-year-old gestational diabetic who has occasional bout of nausea. c. A 55-year-old who has type 2 diabetes mellitus and is adjusting well to the lifestyle changes. d. A 72-year-old who has type 2 diabetes mellitus and is managed with diet and exercise. - correct answera. A 23-year-old who has type 1 diabetes mellitus and is being treated for a tooth abscess. Which of these postoperative complications in the first hour after surgery requires immediate intervention? a. Serous draining on the dressing. b. Swelling of an extremity under a cast. c. Vomiting. d. Dehiscence of a wound. - correct answerd. Dehiscence of a wound. Which of these assessments should a nurse make of a client who had a knee replacement this morning? a. Pain. b. Signs of infection. c. Bowel movement frequency. d. Range of motion. - correct answera. Pain. Which of these actions should a nurse take prior to assisting an elderly client to shave his face? a. Have the client sign a consent form. b. Determine what medications the client takes. c. Soften the client's skin by applying lotion. d. Cleanse the face with a bactericidal solution. - correct answerb. Determine what medications the client takes. Which of these factors should a nurse consider when delegating tasks to unlicensed assistive personnel (UAP)? APEA Predictor with complete answers latest updated 2024 A licensed practical nurse (LPN) is assigned to care for all of these clients. Which client should the nurse assess first? a. A 25-year-old client who is terminally ill with metastatic testicular cancer. b. A 37-year-old client who has second-degree burns on both feet. c. A 49-year-old client who has an acute myocardial infarction related to cocaine ingestion. d. A 68-year-old client who is bed bound related to severe Parkinson's disease. - correct answerc. A 49-year-old client who has an acute myocardial infarction related to cocaine ingestion. Which of these preventative measures should a nurse manager in a long-term care facility plan to institute to decrease clients' risks for falls? a. Monitoring clients frequently for evidence of activity intolerance. b. Placing all client personal items in the bedside drawers. c. Raising the side rails for all clients who have memory impairment. d. Maintaining all client beds in the highest position. - correct answera. Monitoring clients frequently for evidence of activity intolerance. Which of these assessment findings, if present in a primigravida, indicates that the client is experiencing true labor? a. The pains are felt in the lower abdomen, back, and groin. b. The Braxton-Hicks contractions have become stronger and more frequent. c. There is an increased amount of white mucus discharge. d. There is a progressive increase in effacement and cervical dilatation. - correct answerd. There is a progressive increase in effacement and cervical dilatation. A client is admitted for opiate detoxification for the fifth time. Which of these statements, if made by a staff member, indicates a biased view of the client? a. "I feel so frustrated when clients are re-admitted." b. "Addicts relapse because they don't try hard enough." c. "I think this client needs to consider long-term placement after detoxification." d. "The team really needs to discuss this client's treatment plan." - correct answerb. "Addicts relapse because they don't try hard enough." Which of these women, each of whom is in labor, should a nurse recognize as in need of immediate attention? a. A woman who is having contractions every 6 to 8 minutes of mild to moderate intensity. b. A woman who is receiving oxytocin augmentation and who has contractions lasting 60 to 70 seconds. c. A woman who is in the active phase of labor and who insists she needs to use the bedpan to have a bowel movement. d. A woman whose uterine contractions frequency is every two to give minutes. - correct answerc. A woman who is in the active phase of labor and who insists she needs to use the bedpan to have a bowel movement. APEA Predictor with complete answers latest updated 2024 A nurse has received a report on these assigned clients. Which client should the nurse follow-up first? a. A client, admitted with acute diverticulitis, who has a white blood cell count (WBC) of 10,000 mm3. b. A client, admitted with acute pancreatitis, who has a fasting serum glucose of 130 mg/dL today, and had a reading of 160 mg/dL yesterday. c. A client, admitted with hepatitis, who has jaundice and tea-colored urine. d. A client who is currently receiving cancer chemotherapy and who has a white blood cell count of 500 mm3 today. - correct answerd. A client who is currently receiving cancer chemotherapy and who has a white blood cell count of 500 mm3 today. Which of these statements, if made by a client who is taking a diuretic, should a nurse recognize as indicative of the need for additional instructions? a. "I take all of my medications at bedtime so I don't forget them." b. "I eat one or two bananas every day." c. "I weigh myself every day in the morning." d. "I will call my doctor if I have muscle weakness." - correct answera. "I take all of my medications at bedtime so I don't forget them." A nurse is monitoring a client who had a cystoscopy six hours ago. The nurse should inform the physician of which these manifestations? a. The client has pink-tinged urine. b. The client reports burning on urination. c. The client's white blood cell count is 15,000 mm3. d. The client appears drowsy. - correct answerc. The client's white blood cell count is 15,000 mm3. Which of these actions should a nurse perform prior to a client's scheduled hemodialysis? a. Administer prophylactic antibiotics. b. Weigh the client. c. Give the client normal saline solution to drink. d. Measure the urine specific gravity. - correct answerb. Weigh the client. Which of these behaviors, if taken by a staff nurse on a psychiatric unit, indicates a correct understanding of therapeutic techniques? a. A nurse smiles when speaking with clients who are manic. b. A nurse uses touch to communicate concern with a depressed client. c. A nurse sets consistent limits with manipulative clients. d. A nurse shares own anxiety reduction techniques with a client who has panic attacks. - correct answerc. A nurse sets consistent limits with manipulative clients. A client has been in bed for the past three days. Which of these measures should a nurse include before assisting the client out of bed? APEA Predictor with complete answers latest updated 2024 a. Having the client drink a glass of water. b. Raising the head of the bed. c. Flexing the client's knees. d. Assessing the lung sounds. - correct answerb. Raising the head of the bed. A client who has insulin-dependent diabetes mellitus asks a nurse, "What should I do when I feel nervous, sweaty, and hungry?" The nurse should give the client which of these instructions? a. "Lie down and rest." b. "Eat a carbohydrate snack." c. "Take your prn dose of insulin." d. "Add a slice of bread to your next meal." - correct answerb. "Eat a carbohydrate snack." Which of these tasks should a licensed practical nurse (LPN) delegate to a nursing assistant? a. Checking the 11 A.M. blood sugar for a client who has ketoacidosis. b. Measuring the pulse oximetry level for a client who has status asthmaticus. c. AMbulating a client who had a hip replacement three days ago. d. Changing the dressing for a client who had wound debridement last week. - correct answerc. AMbulating a client who had a hip replacement three days ago. A 36-week-pregnant woman awakens to find she is having profuse, red vaginal bleeding. A nurse should prepare the woman to have an immediate sonogram to determine the: a. location of the placenta. b. uterine response to labor. c. the fetus's current weight. d. condition of the uterine vascular bed. - correct answera. location of the placenta. A nurse is planning to interview a client who speaks limited English. Which of these strategies should the nurse include? a. Smile frequently during the interview interview to reduce the client's anxiety. b. Observe the client for indicators of confusion or not understanding questions. c. Maintain constant eye contact throughout the interview. d. Keep the interview short to decrease the client's fatigue. - correct answerb. Observe the client for indicators of confusion or not understanding questions. A nurse takes the weight of a normal 2-year-old child who comes in to the pediatric clinic for a well-child visit. If the child weighted 7 lbs, 2 oz. at birth, how much should the nurse expect the child to weight at this visit? a. 14 lbs, 2 oz. b. 18 lbs, 6 oz. c. 28 lbs, 8 oz. d. 45 lbs, 10 oz. - correct answerc. 28 lbs, 8 oz. APEA Predictor with complete answers latest updated 2024 c. Raising muffin. d. Green salad. - correct answerb. Bran flakes. Which of these discharge instructions should a nurse include for a client who has a ruptured tympanic membrane that occurred during a fall? a. "No showers or washing of the hair for the next month." b. "Avoid yawning or holding your head down." c. "Do not allow any water to enter the ear until healing is confirmed by direct visualization." d. "Avoid swallowing and coughing until your ear has healed." - correct answerc. "Do not allow any water to enter the ear until healing is confirmed by direct visualization." Which of these nursing measures is appropriate for a client who has recurrent renal calculi? a. Weighing the client daily before breakfast. b. Measuring the blood pressure every four hours. c. Encouraging a daily intake of three liters of fluids. d. Testing the urine for protein each shift. - correct answerc. Encouraging a daily intake of three liters of fluids. When auscultating the lungs of a woman who is admitted for severe pregnancy-induced hypertension, a nurse notes the presence of crackles and moist respirations. These assessment findings most likely indicate which of these complications? a. A convulsion is imminent. b. Pulmonary edema has developed. c. Bilateral lobar pneumonia is present. d. Respiratory failure is evident. - correct answerb. Pulmonary edema has developed. A licensed practical nurse (LPN) is assigned to care for all of these clients. Which client should the nurse assess first? a. A client who is eight-hours postoperative after a hip replacement. b. A client who is drowsy after falling out a third story window. c. A client who is four hours post-colonoscopy and polyp removal. d. A client who is dysphasic after a transient ischemic attack. - correct answerb. A client who is drowsy after falling out a third story window. Which of these clients is at the highest risk of developing osteoporosis? a. An obese African-American adolescent who does not exercise. b. A pregnant Asian client who is a vegetarian. c. A middle-aged Native-American male who is quadriplegic. d. A thin, elderly Caucasian female who lives alone. - correct answerd. A thin, elderly Caucasian female who lives alone. APEA Predictor with complete answers latest updated 2024 A nurse is obtaining the health history of a client who is admitted for surgical repair of an inguinal hernia. Which of these factors should the nurse recognize as having the greatest impact on the outcome of the surgery? a. The client takes several acetylsalicylic acid (Aspirin) tablets daily for knee pain. b. The client drinks one glass of beer every evening with dinner. c. The client had a knee replacement six months prior to this admission. d. The client is allergic to all penicillin-type antibiotics. - correct answera. The client takes several acetylsalicylic acid (Aspirin) tablets daily for knee pain. A nurse should recognize that a client who has chronic obstructive pulmonary disease (COPD), needs additional instructions if the client makes which of these statements? a. "I will try to take slow, deep breaths when I feel short of breath." b. "I will use the albuterol (Proventil) nebulizer before I eat. c. "I will drink most of my fluids between meals." d. "I will turn up the oxygen flow rate if I have difficulty breathing." - correct answerd. "I will turn up the oxygen flow rate if I have difficulty breathing." A woman is treated in the emergency room for a broken arm and multiple facial bruises caused by her spouse. Which of these statements, if made by a nurse, is therapeutic? a. "You should leave this relationship now or you will be sorry." b. "Are you aware that women who remain in abusive relationships eventually are killed?" c. "This type of abuse typically recurs after a period of remorse by the abuser." d. "Can you think of what you did to cause this abuse?" - correct answerc. "This type of abuse typically recurs after a period of remorse by the abuser." What is the biggest side effect of colchicine? - correct answerdiarrhea How would you describe the appearance of molluscum contagiosum? - correct answerpapules that are umbilicated and contain a caseous plug How do we treat a broken clavicle in an infant? - correct answerno treatment; it should heal on it's own At what week of pregnancy is the uterus palpable just above the pubic symphysis? - correct answerweek 12 At what week of pregnancy is the fundus palpable halfway between the pubic symphysis and umbilicus? - correct answerweek 16 At what week of pregnancy is the fundus of the uterus at the umbilicus? - correct answerweek 20 At what week of pregnancy is the fundus of the uterus halfway between the xiphoid process and umbilicus? - correct answerweek 28 APEA Predictor with complete answers latest updated 2024 At what week of pregnancy is the fundus just below the xiphoid process? - correct answerweek 34 Increased sweat production is a sign of what endocrine disorder? - correct answerhyperthyroidism What is Paget's disease? - correct answerthere is localized increased bone turnover and blood flow resulting in the breakdown of bone and replacing it with weakened and highly vascular bone putting the indiv at increased risk of fractures How do we treat Paget's diseease? - correct answerbisphosphonates How should pregnant women wear their seatbelt? - correct answerwith the shoulder strap like a normal person and then the groin strap below the belly and across the hips What is another name for fifth disease? - correct answerparvovirus aka slapped cheek disease aka erythema infectiosum Which type of prevention are vaccinations? - correct answerprimary When should patients begin antiretroviral therapy for HIV infection? - correct answeras soon as it is detected, even if in the acute phase What are the first generation antihistamines? - correct answerdiphenhydramine (benadryl) and chlorpeniramine (actifed) What are s/s of the secondary stage of syphilis? - correct answerrash on hands and feet, lymphadenopathy, fever What is the recommended treatment for chronic bacterial prostatitis? - correct answera fluoroquinolone (cipro or levo) + bactrim What is the recommended treatment for acute prostatitis? - correct answercipro (if not STI related) or ceftriaxone What are s/s of an intraductal breast papilloma? - correct answerclear to bloody unilateral nipple discharge (bilateral is usually benign), and also a wart like lump palpated in the nipple area If a patient has GABHS but has an allergy to penicillins, what is the second line option? - correct answerfirst generation cephalosporins, unless the allergy is severe, then you would consider macrolides like a -mycin APEA Predictor with complete answers latest updated 2024 If a patient's Coomb's test is positive, what do we do? - correct answernothing, she is RH positive so we do not need to treat If a patient's Coomb's test is negative, what do we do? - correct answergive rhogam at 28 weeks and 72 hours after birth What is the only form of non-hormonal contraception other than barrier methods such as condoms? - correct answerIUD The vaginal ring for contraception must be taken out how often? - correct answerevery 3 weeks If a patient is on birth control and is currently having breakthrough bleeding, then what do we do? - correct answerincrease the progesterone How do we treat temporal arteritis? - correct answersteroids If a patient is on birth control and they miss one pill, what should they do? - correct answerdouble up If a patient is on birth control and they miss two pills, what should they do? - correct answerdouble up for 2 days If any woman comes in with painless vaginal bleeding between periods (esp if she is postmenopausal), what should you do? - correct answerdo a pregnancy test and then refer for endometrial biopsy to rule out endometrial cancer In terms of the breasts, _____________ is used for screening and _____________ is used for diagnosis. - correct answermammogram; ultrasound When do we screen for group B strep in pregnancy? - correct answerweeks 35-37 When do we perform alpha fetoprotein screening in pregnancy? - correct answer15-20 weeks Alpha fetoprotein screening in pregnancy can help determine the presence of what in pregnancy? - correct answerspina bifida and anencephaly What is placenta previa? - correct answerwhen the placenta blocks the cervical opening of the uterus What is placenta abruptio? - correct answerwhen the placenta either fully or partially detaches from the uterine wall APEA Predictor with complete answers latest updated 2024 When does placental abruption most commonly occur? - correct answerin the third trimester What is the major difference in symptomalogy between placenta abruptio and placenta previa? - correct answera placental abruption is painful whereas placenta previa is painless If a pregnant woman comes in with painless vaginal bleeding, what should we be considering? - correct answerplacenta previa What is the recommended treatment for polymyalgia rheumatica? - correct answersteroids Patients with SLE are going to have what abnormal labwork? - correct answerelevated ESR and CRP How do we treat lupus flares? - correct answersteroids What are common sequelae of fibromyalgia? - correct answerinsomnia and depression What is a great treatment option for fibromyalgia? - correct answeramitriptyline (a TCA) because it helps with chronic pain, sleep, and depression How is fibromyalgia diagnosed? - correct answerwhen at least 8 out of 11 points bilaterally are tender to touch and the symptoms have lasted longer than 3 months What is slipped capital femoral epiphysis? - correct answerwhen the femoral head of the hip is out of place What is the trendelenburg sign and when is it positive? - correct answerwhen the patient is standing straight and the pelvis on the unaffected side drops; this is a sign of a slipped capital femoral epiphysis What is the main characteristic of a slipped capital femoral epiphysis? - correct answerwalking with a limp SCFE is most common in which patient population? - correct answeradolescents What is legg-calve-perthe disease? - correct answeravascular necrosis of the femoral head due to a lack of blood supply Legg-calve-perthes disease is most common in which patient population? - correct answeryoung children What is the radiographic study of choice for the knee? - correct answerMRI APEA Predictor with complete answers latest updated 2024 What is the radiographic study of choice for the back? - correct answerx ray What is bursitis? - correct answera fluid collection in the bursa of a joint How is bursitis treated? - correct answera needle to instill intraarticular steroids and then pull off fluid Bursitis can be a __________ issue - correct answerrecurrent The medical term for knock knees - correct answergenu valgum The medical term for bow legs - correct answergenu varum What is myasthenia gravis? - correct answeran autoimmune disorder that causes muscle weakness of the voluntary muscles Genu valgum is caused by a problem with what? - correct answerthe medial collateral ligament; think valgum as in gum makes your knees stick together Genu varum is caused by a problem with what? - correct answerthe lateral collateral ligament What are shin splints? - correct answera medial tibial syndrome caused by overuse of the muscles How do we treat shin splints? - correct answerrest and avoidance of exercise until the pain resolves What is the tell-tale difference between a herniated disc and sciatica? - correct answersciatica feels better when you stand, a herniated disc feels better when you sit down When will spinal or lumbar stenosis feel better? - correct answerwhen the patient sits down What is Korsakoff syndrome? - correct answera deficiency in thiamine and folate caused by alcoholism What changes to the vagina occur with aging? - correct answerdryness, loss of rugae, less pubic hair What is a navicular fracture? - correct answeralso known as a scaphoid fracture; this impacts the snuff box and causes tenderness there. This will have to be casted to stabilize the fracture APEA Predictor with complete answers latest updated 2024 If a patient's pap results show NILM, what should you do? - correct answerrepeat in 3 years If a patient's pap results show ASC-H or HSIL what should you do? - correct answerrefer for colposcopy The valgus stress test assesses what? - correct answerthe medial collateral ligament The varus stress test assesses what? - correct answerthe lateral collateral ligament The Lachman's test assesses what? - correct answerthe ACL The McMurray's test assesses for what? - correct answera torn meniscus The anterior drawer test assesses what? - correct answerthe acl The posterior drawer test assesses what? - correct answerthe posterior cruciate ligament (pcl) a normal rise in blood sugar as a person's body prepares to wake up due to a release of cortisol and growth hormone - correct answerdawn phenomenon How can you tell between the dawn and somogyi effect? - correct answerIf the blood sugar level is low at 2 a.m. to 3 a.m., suspect the Somogyi effect. If the blood sugar level is normal or high at 2 a.m. to 3 a.m., it's likely the dawn phenomenon. A 14yo male with bronchitis is being treated with fluids and expectorants. He returns to the clinic with a fever of 103F, right pleuritic chest pain, and green sputum. Which of the following examination results would be expected? A. Right lower lobe crackles B. Decreased Fremitus C. Bilateral Wheezing D. Normal Percussion - correct answerA. Right lower lobe crackles. 2 year old child is diagnosed with radial head subluxation ("nursemaid's elbow"). After closed manipulation, the best indication of successful treatment is that: A. Swelling dissipates immediately B. Pulse and sensation are restored C. The child quickly begins to use the affected arm D. A click is felt while the child is extending and rotating the arm - correct answerC. The child quickly begins to use the affected arm. Which of the following would be an appropriate alternative to erythromycin therapy in an 18 year old patient with Mycoplasma pneumoniae infection? A. Azithromycin (Zithromax) APEA Predictor with complete answers latest updated 2024 B. Cephalexin (Keflex) C. Amoxicillin D. Clindamycin (Cleocin) - correct answerA. Azithromycin A 10-month old presents with a rash, runny nose, and cough. Examination reveals a cluster of tiny white papuls with an erythematous base on the buccal mucosa. What does this suggest? A. Scarlet fever B. Rubella C. Erythema infectiosum D. Measles - correct answerD. Measles Office Spriometry performed with an albuterol nebulizer treatment can confirm a diagnosis of asthma because it indicates: A. Oxygen saturation B. Whether airway obstruction is from pulmonary fibrosis C. Chronic carbon dioxide retention D. Whether a patient has reversible airway obstruction - correct answerD. Whether a patient has reversible airway obstruction An 18yr old patient presents with complaints of maxillary facial pain and yellow nasal discharge for 14 days. What is the appropriate initial pharmacologic intervention? A. Amoxicillin-Clavulanate (Augmentin) B. Ceftriaxone (Rocephin) C. Levofloxacin (Levaquin) D. Erythromycin - correct answerA. Amoxicillin-Clavulanate (Augmentin) Which of the following is the leading cause of cancer-related deaths in the majority of women? A. cervical Cancer B. Ovarian Cancer C. Breat Cancer D. Lung Cancer - correct answerD. Lung Cancer A 29-year old male with noncomplicated Chlamidia infection may exhibit: A. Urticaria B. No remarkable clinical symptoms C. A green mucoid penile discharge D. A penile ulcer - correct answerB. No remarkable clinical symptoms To assess a patient's ability to think abstractly, a nurse practitioner could ask the patient: A. The meaning of a common proverb B. What action the patient would take if their was a fire in the house C. To count backwards from 100 by 7s APEA Predictor with complete answers latest updated 2024 D. To spell a word backward. - correct answerA. The meaning of a common proverb A common rule to follow when prescribing many medications for the elderly is to: A. Give most medications in the morning to allow slowed body functions to have time to adjust to the drug. B. Start at a lower dose than what is commonly prescribed for adults, and increase the dose slowly. C. Perform a creatinine clearance test for baseline data on all person over age 70 before starting on new medications. D. Give medication with a full glass of water to promote absorption and limit harm to the kidneys. - correct answerB. Start at a low dose than what is commonly prescribed for adults, and increase the dose slowly. A 72 year old female presents with a medical history unremarkable except for the occasional headache. She enjoyed emotional health throughout marriage and childrearing, although her family history is significant for strokes and nervous breakdowns. Since her husband died a year ago, however, she has been waking every morning at 3am, is reluctant to go out although she has family member living nearby, and feels a continual urge to sleep. She cannot discuss her late husband without crying. Differential diagnoses would include: A. Stroke, depression, and hyperthyroidism B. Depression, Unresolved grief, and hypothyroidism C. Migraine, anxiety disorder, and hypertension D. Dementia, delirium, and pna - correct answerB. Depression, Unresolved grief, and hypothyroidism The parent of a 13-year old diagnosed with seizure disorder calls to report that the child is exhibiting symptoms of a cold, but has no fever. The nurse practitioner should advise the parent that development of fever may: A. make the seizure medication less effective B. Lower the seizure threshold C. Increase the seizure threshold D. Have no effect on the seizure threshold - correct answerB. Lower the seizure threshold After a 3-week camping trip, an 11-year old is seen for a target lesion with central clearing, located in the inguinal area. The patient has had a severe headache, malaise, fatigue, and generalized musculoskeletal pain for several days. Pharmacologic management of this condition includes: A. trimethoprim-sulamethoxazole (Bactrim) B. Azithromycin (Zithromax) C. Metronidazole (Flagyl) D. Doxycycline (Doryx) - correct answerD. Doxycycline APEA Predictor with complete answers latest updated 2024 An 88 year old patient presents with right-sided weakness after being unable to rise unassisted following a fall to the bathroom floor. History includes aphasia and noncompliance with hypertension medication regimen. What is the most likely diagnosis? A. Seizure B. Right-sided Stroke C. Left-sided stroke D. Trans ischemic attack - correct answerC. Left-sided Stroke A nurse pracitioner is evaluating a 40 year old patient suspected of having a pulmonary embolus. The patient complains of anxiety and cough. A stat chest x-ray is normal. Which of the following tests should the nurse practitioner perform next? A. Spirometry B. MRI C. Contrast venography D. Helical CT pulmonary angiography - correct answerD. Helical CT pulmonary angiography A routine laboratory assessment of a 12 year old patient with a family history of thalassemia and anemia reveals Tanner II presentation and Hct=355. In addition to a complete blood count (CBC), the nurse practitioner should order which of the following? A. serum folic acid B. Vitamin B12 level C. Hemoglobin Electrophoresis D. 24-hour urine creatinine - correct answerC. Hemoglobin electrophoresis A patient who is 28 weeks pregnant reports a single episode of vaginal bleeding. History indicates normal prenatal progress to date, and the patient denies pain, vaginal itching, or discharge. Which of the following is the most appropriate intervention to aid in the diagnosis of this case? A. Nitrazine test B. Non-Stress test C. Ultrasound D. Bimanual cervical exam - correct answerC. Ultrasound The common bacteria responsible for pneumonia in older adults residing in the community is: A. Staphylococcus aureus B. Streptococcus pneumoniae C. Pseudomonas Aeruginosa D. Moraxelle catarrhalis - correct answerB. Strep Pneumo A 40 year old male patient presents with a 2 week history of rectal pain and itching. He reports a past history of constipation, and finding spots of bright red blood on toilet APEA Predictor with complete answers latest updated 2024 paper several times a week. Rectal exam reveals a tender, swollen, bluish, ovoid mass. The stool guaiac test is negative. Which of the following actions should the nurse practitioner take? A. Prescribe diphenoxylate/atropine( Lomotil) B. Schedule a colonoscopy to rule out colon cancer C. Repeat the guaiac test three times and obtain complete blood count D. Prescribe bulk forming agents and hydrocortisone suppositories. - correct answerD. Prescribe bulk forming agents and hydrocortisone suppositories. A 17 year old female is suspected of having polycystic ovary syndrome. In addition to testosterone, the most appropriate diagnostic tests to order would be: A. BUN, creatinine, electrolytes, and dehydroepiandrosterone(DHEA) B. CBC, BUN, creatinine, and estrone C. Fasting blood sugar (FBS), complete blood count (CBC), BUN, and sex hormone binding globulin D. Follicle-stimulating hormone (FSH), Luteinizing hormone (LH), prolactin, and thyroid- stimulating hormone (TSH) - correct answerD. FSH, LH, TSH Which of the following wet-mount results confirms a preliminary diagnosis of bacterial vaginosis? A. Squamous epithelial cells with stippling appearance and indistinct borders, no lactobacillus rods, and many white cells B. Squamous epithelial cells with clear cytoplasm and distinct borders, many lactobacillus rods, and occasional WBC C. Organisms about the size of white blood cells with undulating flagellum, occasional lactobacillus rods, and many WBC D. Hyphae and spores, few lactobacillus rods, and occasional WBC - correct answerSquamous epithelial cells with stippling appearance and indistinct borders, no lactobacillus rods, and many white cells. A patient is referred with a diagnosis of diabetes mellitus, hypertension, and coronary artery disease. The patient is on both insulin and a beta blocker. Assuming that the patient will continue the beta blocker, it will be important to educate the patient on the recognition of hypoglycemia. Which symptom would be most indicative of hypoglycemia in this patient? A. edema B. Tachycardia C. Palpitations D. Sweating - correct answerD. Sweating A 67-year old with type 2 diabetes mellitus, congestive heart failure, and mild coronary artery disease is currently taking Digoxin, 0.25mg daily; HCTZ, 25mg daily; Metformin 500mg daily; glipizide 10mg daily; and Atorvastatin (Lipitor) 20mg HS. Which of the following is an accurate statement regarding the regimen? A. The glipizide will increase the risk of potassium depletion. APEA Predictor with complete answers latest updated 2024 B. The digoxin will increase the risk of hypoglycemia C. The HCTZ will predispose the patient to Digoxin toxicity D. The atorvastatin will worsen the diabetes. - correct answerC. The HCTZ will predispose the patient to Digoxin toxicity The initial therapy for a 3 week old infant with uncomplicated symptoms of gastroesophageal reflux disease includes of the following positions and change in diet? A. Prone position and rotate formulas in order to find one that causes less reflux B. Smaller, more frequent feedings while holding infant C. Semisupine position and offer more ounces of formula less frequently D. Lateral position and start small amounts of solid food with formula - correct answerSmaller, more frequent feedings while holding infant. Which of the following best describes psoriatic lesions in an elderly patient? A. Shiny, purple, smooth lesions B. Localized erythematous vesicles C. Erythematous plagues with central clearing D. Red, sharply defined plaques with silvery scales - correct answerD. Red, sharply defined plaques with silvery scales A 68-yo female presents with a new onset of left-sided throbbing headache. She has noticed some spots in her visual fields that come and go. She is being treated with an NSAID for arthritis. Sedimentation rate is elevated, but all other lab values are WNL. The headache is most likely due to: A. TIA B. NSAID induced headache C. Temporal arteritis D. Glaucoma - correct answerC. Temporal arteritis A patient with Type 1 diabetes mellitus who is on NPH and regular insulin split-dosing presents with complaints of early morning rise in fingerstick blood glucose. A review of an at-home glucose test reveals increased morning levels. After an increase in the evening insulin dose, the problem worsens. This is most likely an example of: A. Insulin resistance B. Cortisol deficiency C. the Somogyi effect D. pawn phenomenon - correct answerC. the Somogyi effect An 18 yo college student presents for an athletic physical. When asked about current medications, she mentions that she takes "some hearb" she bought at a health food store for migrains and menstrual cramps. Which of the following herbal remedies has been commonly used for these conditions? A. Ephedra B. Echinacea C. Feverfew APEA Predictor with complete answers latest updated 2024 D. jaw pain extending to the neck - correct answerA. Electric-shock-like unilateral facial pain A 65-yo male presents to a clinic complaining of increasing fatigue, dyspnea on exertion, and ankle edema during the day. He has a history of mild hypertension, for which he saw his physician years ago. The physician advised the patient to decrease his salt intake. On physical exam, the patient is tachycardic, positive for jugular venous distension, and positive for S3, with a systolic murmur. Chest xray reveals cardiomegaly with vascular redistribution. The nurse practitioner's diagnosis should be: A. mycardial infarction B. pneumonia C. Heart failure D. pleural effusion - correct answerC. Heart failure An 88-yo patients has had a gradual onset of hearing loss in the left ear. Examination shows a large accumulation of cerumen in the external auditory canal. Assuming there is no neural loss, the nurse practitioner would expect the Weber test to: A. lateralize to the left B. lateralize to the right C. Be inconclusive D. Not lateralize - correct answerA. Lateralize to the left. A 38-yo male experiences a sudden onset of acute abdominal pain radiating to his groin. He is having difficulty walking and is nauseated. He denies any trauma or previous groin injuries. Examination reveals diffuse swelling of the left testicle and negative cremasteric reflex. The most likely diagnosis is: A. Testicluar cancer B. Testicular torsion C. Appendicitis D. Epididymitis - correct answerB. Testicular Torsion Research findings have shown that, in order to improve the longevity of a patient who has COPD, the treatment of choice is: A. oxygen B. Anticholinergic drugs C. Systemic steroids D. Exercise - correct answerA. Oxygen A 15 year old patients returns for contraceptive services 2 weeks after a diagnosis of trichomonas vaginitis and treatment with 2gm of metronidazole (Flagyl). She reports that discharge and itching are gone, but she is urinating frequently, accompanied with alot of burning. The patient has not resumed sexual activity and has menstrated since her last visit. Examination reveals mild suprapubic tenderness, no leukorrhea, and a normal wet mount. Gonococcal culture and chlamydia tests are negative. Which diagnostic test should be performed immediately? APEA Predictor with complete answers latest updated 2024 A. Urine pregnancy test B. Microscopic exam of urine C. Herpes simplex virus culture D. Repeat chlamydia test. - correct answerB. Microscopic exam of urine. 14 year old patient who fell on a outstretched hand complains of proximal forearm pain. X-ray reveals a positive fat pad sign, and the patient is unable to fully extend the elbow. No definitive bony changes are seen on xray. The most likely working diagnosis is: A. upper arm sprain B. lateral epicondylitis C. Radial head fracture D. Olecranon bursitis - correct answerC. Radial head fracture. Which of the following would be most appropriate to perform in the initial evaluation of a patient with symptoms of acute prostatitis? A. Urinalysis and urine culture B. Scrotal palpation and urine culture C. Prostate-specific antigen (PSA) D. Complete blood count with differential (CBC with Diff) - correct answerA. Urinalysis and urine culture Which of the following is the most serious outcome of Barrett's esophagus? A. Esophageal adenocarcinoma B. Biliary atresia C. Liver hemangioma D. Esophageal varices - correct answerA. Esophageal adenocarcinoma A 4-yo girl with a history of recurrent urinary tract infections returns for follow-up after diagnosis of the most recent infection. During the physical exam, the 4 yo tells the nurse practitioner that her brother (age 9) lies on top of her and rubs his penis in her pubic area. After completing the physical exam, the nurse practitioner must: A. Discuss the child's report with the mother B. Report suspicion's to the appropriate child protective services agency C. Refer the case to a physician D. Suggest psychological counseling. - correct answerB. Report suspicions to the appropriate child protective services agency. A 20 yo male presents with a 1 month history of a " bump on my testicle" He reports being sexually active, frequent episodes of unprotected intercourse with multiple partners. Which condition should be suspected? A. Cancer of the testicles B. Torsion of the testicles C. Acute epididymitis D. Gonorrhea - correct answerA. Cancer of the testicles APEA Predictor with complete answers latest updated 2024 Which class of antihypertensive agents has been associated with acute renal failure and is contraindicated in patients with bilateral renal artery stenosis? A. Calcium channel blockers B. Thiazide diuretics C. Potassium-sparing diuretics D. Angiotensin-converting enzyme (ACE) inhibitors. - correct answerD. ACE inhibitors A 70 yo male presents with parathesia of the lower extremities. On exam, the patient appears pale and shows a decreased vibratory sense. Lab tests reveal elevated indirect bilirubin; Hct=30%; Mean corpuscular volume(MCV)=120 (norm 87-103). The most likely diagnosis is: A. pernicious anemia B. Anemia of chronic disease C. folic acid deficiency D. Iron deficiency anemia - correct answerA. Pernicious anemia Which of the following findings is typically a sign of acute appendicitis? A. Positive Murphy's sign B. Rebound tenderness at McBurney's point C. A negative Rovsing's sign D. Positive Prehn's sign - correct answerRebound tenderness at McBurney's point Which of the following signs/symptoms are often associated with headaches due to an intracranial tumor? A. Pain worse in supine position; focal neurological signs B. Hyperreflexia; personality changes C. Acute onset; increasing pain over hours to days D. Pupillary constriction; stupor - correct answerA. Pain worse in supine position; focal neurological signs A 65 yo female presents with shoulder and pelvic girdle pain for the past 6 months. She reports recent unintentional weight loss. On physical exam, there is pain on ROM, with no weakness noted. Lab studies show a low hemoglobin and elevated SED rate. Which of the following is the most likely diagnosis? A. Polymyositis B. Osteoarthrits C. Polymyagia Rheumatica D. Fibromyalgia - correct answerPolymyalgia Rheumatica A 39 yo patient was diagnosed with acute bronchitis in the emergency department and treated with acetaminophen, dextromethorphan, and metaproterenol (Alupent). The patient's history reveals a smoking habit of 1 pack per day. The patient now presents to a nurse practitioner's office with a fever of 101.2 and a cough productive of thick, yellow- green, foul-smelling sputum. The nurse practitioner should encourage smoking cessation and prescribe:
Docsity logo



Copyright © 2024 Ladybird Srl - Via Leonardo da Vinci 16, 10126, Torino, Italy - VAT 10816460017 - All rights reserved